Programs & Examples On #Abstract function

Abstract functions are functions that can not be used. They exist to provide an interface specification to several concrete classes.

What is (functional) reactive programming?

In pure functional programming, there are no side-effects. For many types of software (for example, anything with user interaction) side-effects are necessary at some level.

One way to get side-effect like behavior while still retaining a functional style is to use functional reactive programming. This is the combination of functional programming, and reactive programming. (The Wikipedia article you linked to is about the latter.)

The basic idea behind reactive programming is that there are certain datatypes that represent a value "over time". Computations that involve these changing-over-time values will themselves have values that change over time.

For example, you could represent the mouse coordinates as a pair of integer-over-time values. Let's say we had something like (this is pseudo-code):

x = <mouse-x>;
y = <mouse-y>;

At any moment in time, x and y would have the coordinates of the mouse. Unlike non-reactive programming, we only need to make this assignment once, and the x and y variables will stay "up to date" automatically. This is why reactive programming and functional programming work so well together: reactive programming removes the need to mutate variables while still letting you do a lot of what you could accomplish with variable mutations.

If we then do some computations based on this the resulting values will also be values that change over time. For example:

minX = x - 16;
minY = y - 16;
maxX = x + 16;
maxY = y + 16;

In this example, minX will always be 16 less than the x coordinate of the mouse pointer. With reactive-aware libraries you could then say something like:

rectangle(minX, minY, maxX, maxY)

And a 32x32 box will be drawn around the mouse pointer and will track it wherever it moves.

Here is a pretty good paper on functional reactive programming.

Disable pasting text into HTML form

Just got this, we can achieve it using onpaste:"return false", thanks to: http://sumtips.com/2011/11/prevent-copy-cut-paste-text-field.html

We have various other options available as listed below.

<input type="text" onselectstart="return false" onpaste="return false;" onCopy="return false" onCut="return false" onDrag="return false" onDrop="return false" autocomplete=off/><br>

How to change font-size of a tag using inline css?

use this attribute in style

font-size: 11px !important;//your font size

by !important it override your css

Convert number of minutes into hours & minutes using PHP

Thanks to @Martin_Bean and @Mihail Velikov answers. I just took their answer snippet and added some modifications to check,

  1. If only Hours only available and minutes value empty, then it will display only hours.

  2. Same if only Minutes only available and hours value empty, then it will display only minutes.

  3. If minutes = 60, then it will display as 1 hour. Same if minute = 1, the output will be 1 minute.

Changes and edits are welcomed. Thanks. Here is the code.

function convertToHoursMins($time) {

            $hours    = floor($time / 60);
            $minutes  = ($time % 60);


            if($minutes == 0){

                if($hours == 1){

                    $output_format = '%02d hour ';

                }else{

                    $output_format = '%02d hours ';
                }


                $hoursToMinutes = sprintf($output_format, $hours);

            }else if($hours == 0){

                if ($minutes < 10) {
                        $minutes = '0' . $minutes;
                }

                if($minutes == 1){

                    $output_format  = ' %02d minute ';

                }else{

                    $output_format  = ' %02d minutes ';
                }

                $hoursToMinutes = sprintf($output_format,  $minutes);

            }else {

                if($hours == 1){

                    $output_format = '%02d hour %02d minutes';

                }else{

                    $output_format = '%02d hours %02d minutes';
                }

                $hoursToMinutes = sprintf($output_format, $hours, $minutes);
            }

            return $hoursToMinutes;
        }`

font awesome icon in select option

If you want the caret down symbol, remove the "appearence: none" it implies to remove webkit and moz- as well from select in css.

PHP header redirect 301 - what are the implications?

The effect of the 301 would be that the search engines will index /option-a instead of /option-x. Which is probably a good thing since /option-x is not reachable for the search index and thus could have a positive effect on the index. Only if you use this wisely ;-)

After the redirect put exit(); to stop the rest of the script to execute

header("HTTP/1.1 301 Moved Permanently"); 
header("Location: /option-a"); 
exit();

Can't push to remote branch, cannot be resolved to branch

Based on my own testing and the OP's comments, I think at some point they goofed on the casing of the branch name.

First, I believe the OP is on a case insensitive operating system like OS X or Windows. Then they did something like this...

$ git checkout -b SQLMigration/ReportFixes
Switched to a new branch 'SQLMigration/ReportFixes'

$ git push origin SqlMigration/ReportFixes
fatal: SqlMigration/ReportFixes cannot be resolved to branch.

Note the casing difference. Also note the error is very different from if you just typo the name.

$ git push origin SQLMigration/ReportFixme
error: src refspec SQLMigration/ReportFixme does not match any.
error: failed to push some refs to '[email protected]:schwern/testing123.git'

Because Github uses the filesystem to store branch names, it tries to open .git/refs/heads/SqlMigration/ReportFixes. Because the filesystem is case insensitive it successfully opens .git/refs/heads/SqlMigration/ReportFixes but gets confused when it tries to compare the branch names case-sensitively and they don't match.

How they got into a state where the local branch is SQLMigration/ReportFixes and the remote branch is SqlMigration/ReportFixes I'm not sure. I don't believe Github messed with the remote branch name. Simplest explanation is someone else with push access changed the remote branch name. Otherwise, at some point they did something which managed to create the remote with the typo. If they check their shell history, perhaps with history | grep -i sqlmigration/reportfixes they might be able to find a command where they mistyped the casing.

Query an XDocument for elements by name at any depth

I am using XPathSelectElements extension method which works in the same way to XmlDocument.SelectNodes method:

using System;
using System.Xml.Linq;
using System.Xml.XPath; // for XPathSelectElements

namespace testconsoleApp
{
    class Program
    {
        static void Main(string[] args)
        {
            XDocument xdoc = XDocument.Parse(
                @"<root>
                    <child>
                        <name>john</name>
                    </child>
                    <child>
                        <name>fred</name>
                    </child>
                    <child>
                        <name>mark</name>
                    </child>
                 </root>");

            foreach (var childElem in xdoc.XPathSelectElements("//child"))
            {
                string childName = childElem.Element("name").Value;
                Console.WriteLine(childName);
            }
        }
    }
}

Remove x-axis label/text in chart.js

If you want the labels to be retained for the tooltip, but not displayed below the bars the following hack might be useful. I made this change for use on an private intranet application and have not tested it for efficiency or side-effects, but it did what I needed.

At about line 71 in chart.js add a property to hide the bar labels:

// Boolean - Whether to show x-axis labels
barShowLabels: true,

At about line 1500 use that property to suppress changing this.endPoint (it seems that other portions of the calculation code are needed as chunks of the chart disappeared or were rendered incorrectly if I disabled anything more than this line).

if (this.xLabelRotation > 0) {
    if (this.ctx.barShowLabels) {
        this.endPoint -= Math.sin(toRadians(this.xLabelRotation)) * originalLabelWidth + 3;
    } else {
        // don't change this.endPoint
    }
}

At about line 1644 use the property to suppress the label rendering:

if (ctx.barShowLabels) {    
    ctx.fillText(label, 0, 0);
}

I'd like to make this change to the Chart.js source but aren't that familiar with git and don't have the time to test rigorously so would rather avoid breaking anything.

ORA-01008: not all variables bound. They are bound

I'd a similar problem in a legacy application, but de "--" was string parameter.

Ex.:

Dim cmd As New OracleCommand("INSERT INTO USER (name, address, photo) VALUES ('User1', '--', :photo)", oracleConnection)
Dim fs As IO.FileStream = New IO.FileStream("c:\img.jpg", IO.FileMode.Open)
Dim br As New IO.BinaryReader(fs)
cmd.Parameters.Add(New OracleParameter("photo", OracleDbType.Blob)).Value = br.ReadBytes(fs.Length)
cmd.ExecuteNonQuery() 'here throws ORA-01008

Changing address parameter value '--' to '00' or other thing, works.

Git push existing repo to a new and different remote repo server?

I have had the same problem.

In my case, since I have the original repository in my local machine, I have made a copy in a new folder without any hidden file (.git, .gitignore).

Finally I have added the .gitignore file to the new created folder.

Then I have created and added the new repository from the local path (in my case using GitHub Desktop).

Preventing multiple clicks on button

One way you do this is set a counter and if number exceeds the certain number return false. easy as this.

var mybutton_counter=0;
$("#mybutton").on('click', function(e){
    if (mybutton_counter>0){return false;} //you can set the number to any
    //your call
     mybutton_counter++; //incremental
});

make sure, if statement is on top of your call.

How to get current instance name from T-SQL

I found this:

EXECUTE xp_regread
        @rootkey = 'HKEY_LOCAL_MACHINE',
        @key = 'SOFTWARE\Microsoft\Microsoft SQL Server',
        @value_name = 'InstalledInstances'

That will give you list of all instances installed in your server.


The ServerName property of the SERVERPROPERTY function and @@SERVERNAME return similar information. The ServerName property provides the Windows server and instance name that together make up the unique server instance. @@SERVERNAME provides the currently configured local server name.

And Microsoft example for current server is:

SELECT CONVERT(sysname, SERVERPROPERTY('servername'));

This scenario is useful when there are multiple instances of SQL Server installed on a Windows server, and the client must open another connection to the same instance used by the current connection.

How to develop or migrate apps for iPhone 5 screen resolution?

First show this image. In that image you show warning for Retina 4 support so click on this warning and click on add so your Retina 4 splash screen automatically add in your project.

Show image here

and after you use this code :

if([[UIScreen mainScreen] bounds].size.height == 568)
    {
        // For iphone 5
    }
    else
    {
        // For iphone 4 or less
    }

What is /dev/null 2>&1?

Let me explain a bit by bit.

0,1,2

0: standard input
1: standard output
2: standard error

>>

>> in command >> /dev/null 2>&1 appends the command output to /dev/null.

command >> /dev/null 2>&1

  1. After command:
command
=> 1 output on the terminal screen
=> 2 output on the terminal screen
  1. After redirect:
command >> /dev/null
=> 1 output to /dev/null
=> 2 output on the terminal screen
  1. After /dev/null 2>&1
command >> /dev/null 2>&1
=> 1 output to /dev/null
=> 2 output is redirected to 1 which is now to /dev/null

Why is SQL server throwing this error: Cannot insert the value NULL into column 'id'?

In my case,

I was trying to update my model by making a foreign key required, but the database had "null" data in it already in some columns from previously entered data. So every time i run update-database...i got the error.

I SOLVED it by manually deleting from the database all rows that had null in the column i was making required.

SSL certificate rejected trying to access GitHub over HTTPS behind firewall

If all you want to do is just to use the Cygwin git client with github.com, there is a much simpler way without having to go through the hassle of downloading, extracting, converting, splitting cert files. Proceed as follows (I'm assuming Windows XP with Cygwin and Firefox)

  1. In Firefox, go to the github page (any)
  2. click on the github icon on the address bar to display the certificate
  3. Click through "more information" -> "display certificate" --> "details" and select each node in the hierarchy beginning with the uppermost one; for each of them click on "Export" and select the PEM format:
    • GTECyberTrustGlobalRoot.pem
    • DigiCertHighAssuranceEVRootCA.pem
    • DigiCertHighAssuranceEVCA-1.pem
    • github.com.pem
  4. Save the above files somewhere in your local drive, change the extension to .pem and move them to /usr/ssl/certs in your Cygwin installation (Windows: c:\cygwin\ssl\certs )
  5. (optional) Run c_reshash from the bash.

That's it.

Of course this only installs one cert hierarchy, the one you need for github. You can of course use this method with any other site without the need to install 200 certs of sites you don't (necessarily) trust.

javascript - replace dash (hyphen) with a space

Imagine you end up with double dashes, and want to replace them with a single character and not doubles of the replace character. You can just use array split and array filter and array join.

var str = "This-is---a--news-----item----";

Then to replace all dashes with single spaces, you could do this:

var newStr = str.split('-').filter(function(item) {
  item = item ? item.replace(/-/g, ''): item
  return item;
}).join(' ');

Now if the string contains double dashes, like '----' then array split will produce an element with 3 dashes in it (because it split on the first dash). So by using this line:

item = item ? item.replace(/-/g, ''): item

The filter method removes those extra dashes so the element will be ignored on the filter iteration. The above line also accounts for if item is already an empty element so it doesn't crash on item.replace.

Then when your string join runs on the filtered elements, you end up with this output:

"This is a news item"

Now if you were using something like knockout.js where you can have computer observables. You could create a computed observable to always calculate "newStr" when "str" changes so you'd always have a version of the string with no dashes even if you change the value of the original input string. Basically they are bound together. I'm sure other JS frameworks can do similar things.

Android Studio Checkout Github Error "CreateProcess=2" (Windows)

I had this issue on Mac. I simply quit Android Studio and restarted it, and for some reason had no further issues.

collapse cell in jupyter notebook

As others have mentioned, you can do this via nbextensions. I wanted to give the brief explanation of what I did, which was quick and easy:

To enable collabsible headings: In your terminal, enable/install Jupyter Notebook Extensions by first entering:

pip install jupyter_contrib_nbextensions

Then, enter:

jupyter contrib nbextension install

Re-open Jupyter Notebook. Go to "Edit" tab, and select "nbextensions config". Un-check box directly under title "Configurable nbextensions", then select "collapsible headings".

How to create a temporary directory and get the path / file name in Python

Use the mkdtemp() function from the tempfile module:

import tempfile
import shutil

dirpath = tempfile.mkdtemp()
# ... do stuff with dirpath
shutil.rmtree(dirpath)

OSError [Errno 22] invalid argument when use open() in Python

you should add one more "/" in the last "/" of path, that is: open('C:\Python34\book.csv') to open('C:\Python34\\book.csv'). For example:

import csv
with open('C:\Python34\\book.csv', newline='') as csvfile:
    spamreader = csv.reader(csvfile, delimiter='', quotechar='|')
    for row in spamreader:
        print(row)

Check Whether a User Exists

Using sed:

username="alice"
if [ `sed -n "/^$username/p" /etc/passwd` ]
then
    echo "User [$username] already exists"
else
    echo "User [$username] doesn't exist"
fi

How can I use optional parameters in a T-SQL stored procedure?

You can do in the following case,

CREATE PROCEDURE spDoSearch
   @FirstName varchar(25) = null,
   @LastName varchar(25) = null,
   @Title varchar(25) = null
AS
  BEGIN
      SELECT ID, FirstName, LastName, Title
      FROM tblUsers
      WHERE
        (@FirstName IS NULL OR FirstName = @FirstName) AND
        (@LastNameName IS NULL OR LastName = @LastName) AND
        (@Title IS NULL OR Title = @Title)
END

however depend on data sometimes better create dynamic query and execute them.

Core dumped, but core file is not in the current directory?

I'm on Linux Mint 19 (Ubuntu 18 based). I wanted to have coredump files in current folder. I had to do two things:

  1. Change /proc/sys/kernel/core_pattern (by # echo "core.%p.%s.%c.%d.%P > /proc/sys/kernel/core_pattern or by # sysctl -w kernel.core_pattern=core.%p.%s.%c.%d.%P)
  2. Raising limit for core file size by $ ulimit -c unlimited

That was written already in the answers, but I wrote to summarize succinctly. Interestingly changing limit did not require root privileges (as per https://askubuntu.com/questions/162229/how-do-i-increase-the-open-files-limit-for-a-non-root-user non-root can only lower the limit, so that was unexpected - comments about it are welcome).

How to get Linux console window width in Python

@reannual's answer works well, but there's an issue with it: os.popen is now deprecated. The subprocess module should be used instead, so here's a version of @reannual's code that uses subprocess and directly answers the question (by giving the column width directly as an int:

import subprocess

columns = int(subprocess.check_output(['stty', 'size']).split()[1])

Tested on OS X 10.9

Difference between "on-heap" and "off-heap"

The heap is the place in memory where your dynamically allocated objects live. If you used new then it's on the heap. That's as opposed to stack space, which is where the function stack lives. If you have a local variable then that reference is on the stack. Java's heap is subject to garbage collection and the objects are usable directly.

EHCache's off-heap storage takes your regular object off the heap, serializes it, and stores it as bytes in a chunk of memory that EHCache manages. It's like storing it to disk but it's still in RAM. The objects are not directly usable in this state, they have to be deserialized first. Also not subject to garbage collection.

Getting vertical gridlines to appear in line plot in matplotlib

Short answer (read below for more info):

ax.grid(axis='both', which='both')

enter image description here

What you do is correct and it should work.

However, since the X axis in your example is a DateTime axis the Major tick-marks (most probably) are appearing only at the both ends of the X axis. The other visible tick-marks are Minor tick-marks.

The ax.grid() method, by default, draws grid lines on Major tick-marks. Therefore, nothing appears in your plot.

Use the code below to highlight the tick-marks. Majors will be Blue while Minors are Red.

ax.tick_params(which='both', width=3)
ax.tick_params(which='major', length=20, color='b')
ax.tick_params(which='minor', length=10, color='r')

Now to force the grid lines to be appear also on the Minor tick-marks, pass the which='minor' to the method:

ax.grid(b=True, which='minor', axis='x', color='#000000', linestyle='--')

or simply use which='both' to draw both Major and Minor grid lines. And this a more elegant grid line:

ax.grid(b=True, which='minor', axis='both', color='#888888', linestyle='--')
ax.grid(b=True, which='major', axis='both', color='#000000', linestyle='-')

What does -save-dev mean in npm install grunt --save-dev

--save-dev: Package will appear in your devDependencies.

According to the npm install docs.

If someone is planning on downloading and using your module in their program, then they probably don't want or need to download and build the external test or documentation framework that you use.

In other words, when you run npm install, your project's devDependencies will be installed, but the devDependencies for any packages that your app depends on will not be installed; further, other apps having your app as a dependency need not install your devDependencies. Such modules should only be needed when developing the app (eg grunt, mocha etc).

According to the package.json docs

Edit: Attempt at visualising what npm install does:

  • yourproject
    • dependency installed
      • dependency installed
        • dependency installed
        • devDependency NOT installed
      • devDependency NOT installed
    • devDependency installed
      • dependency installed
      • devDependency NOT installed

Matplotlib scatterplot; colour as a function of a third variable

There's no need to manually set the colors. Instead, specify a grayscale colormap...

import numpy as np
import matplotlib.pyplot as plt

# Generate data...
x = np.random.random(10)
y = np.random.random(10)

# Plot...
plt.scatter(x, y, c=y, s=500)
plt.gray()

plt.show()

enter image description here

Or, if you'd prefer a wider range of colormaps, you can also specify the cmap kwarg to scatter. To use the reversed version of any of these, just specify the "_r" version of any of them. E.g. gray_r instead of gray. There are several different grayscale colormaps pre-made (e.g. gray, gist_yarg, binary, etc).

import matplotlib.pyplot as plt
import numpy as np

# Generate data...
x = np.random.random(10)
y = np.random.random(10)

plt.scatter(x, y, c=y, s=500, cmap='gray')
plt.show()

How to convert a HTMLElement to a string

You can get the 'outer-html' by cloning the element, adding it to an empty,'offstage' container, and reading the container's innerHTML.

This example takes an optional second parameter.

Call document.getHTML(element, true) to include the element's descendents.

document.getHTML= function(who, deep){
    if(!who || !who.tagName) return '';
    var txt, ax, el= document.createElement("div");
    el.appendChild(who.cloneNode(false));
    txt= el.innerHTML;
    if(deep){
        ax= txt.indexOf('>')+1;
        txt= txt.substring(0, ax)+who.innerHTML+ txt.substring(ax);
    }
    el= null;
    return txt;
}

How to load a controller from another controller in codeigniter?

I came here because I needed to create a {{ render() }} function in Twig, to simulate Symfony2's behaviour. Rendering controllers from view is really cool to display independant widgets or ajax-reloadable stuffs.

Even if you're not a Twig user, you can still take this helper and use it as you want in your views to render a controller, using <?php echo twig_render('welcome/index', $param1, $param2, $_); ?>. This will echo everything your controller outputted.

Here it is:

helpers/twig_helper.php

<?php

if (!function_exists('twig_render'))
{

    function twig_render()
    {
        $args = func_get_args();
        $route = array_shift($args);
        $controller = APPPATH . 'controllers/' . substr($route, 0, strrpos($route, '/'));

        $explode = explode('/', $route);
        if (count($explode) < 2)
        {
            show_error("twig_render: A twig route is made from format: path/to/controller/action.");
        }

        if (!is_file($controller . '.php'))
        {
            show_error("twig_render: Controller not found: {$controller}");
        }
        if (!is_readable($controller . '.php'))
        {
            show_error("twig_render: Controller not readable: {$controller}");
        }
        require_once($controller . '.php');

        $class = ucfirst(reset(array_slice($explode, count($explode) - 2, 1)));
        if (!class_exists($class))
        {
            show_error("twig_render: Controller file exists, but class not found inside: {$class}");
        }

        $object = new $class();
        if (!($object instanceof CI_Controller))
        {
            show_error("twig_render: Class {$class} is not an instance of CI_Controller");
        }

        $method = $explode[count($explode) - 1];
        if (!method_exists($object, $method))
        {
            show_error("twig_render: Controller method not found: {$method}");
        }

        if (!is_callable(array($object, $method)))
        {
            show_error("twig_render: Controller method not visible: {$method}");
        }

        call_user_func_array(array($object, $method), $args);

        $ci = &get_instance();
        return $ci->output->get_output();
    }

}

Specific for Twig users (adapt this code to your Twig implementation):

libraries/Twig.php

$this->_twig_env->addFunction('render', new Twig_Function_Function('twig_render'));

Usage

{{ render('welcome/index', param1, param2, ...) }}

How do I remove a comma off the end of a string?

A simple regular expression would work

$string = preg_replace("/,$/", "", $string)

jQuery Mobile: document ready vs. page events

jQuery Mobile 1.4 Update:

My original article was intended for old way of page handling, basically everything before jQuery Mobile 1.4. Old way of handling is now deprecated and it will stay active until (including) jQuery Mobile 1.5, so you can still use everything mentioned below, at least until next year and jQuery Mobile 1.6.

Old events, including pageinit don't exist any more, they are replaced with pagecontainer widget. Pageinit is erased completely and you can use pagecreate instead, that event stayed the same and its not going to be changed.

If you are interested in new way of page event handling take a look here, in any other case feel free to continue with this article. You should read this answer even if you are using jQuery Mobile 1.4 +, it goes beyond page events so you will probably find a lot of useful information.

Older content:

This article can also be found as a part of my blog HERE.

$(document).on('pageinit') vs $(document).ready()

The first thing you learn in jQuery is to call code inside the $(document).ready() function so everything will execute as soon as the DOM is loaded. However, in jQuery Mobile, Ajax is used to load the contents of each page into the DOM as you navigate. Because of this $(document).ready() will trigger before your first page is loaded and every code intended for page manipulation will be executed after a page refresh. This can be a very subtle bug. On some systems it may appear that it works fine, but on others it may cause erratic, difficult to repeat weirdness to occur.

Classic jQuery syntax:

$(document).ready(function() {

});

To solve this problem (and trust me this is a problem) jQuery Mobile developers created page events. In a nutshell page events are events triggered in a particular point of page execution. One of those page events is a pageinit event and we can use it like this:

$(document).on('pageinit', function() {

});

We can go even further and use a page id instead of document selector. Let's say we have jQuery Mobile page with an id index:

<div data-role="page" id="index">
    <div data-theme="a" data-role="header">
        <h3>
            First Page
        </h3>
        <a href="#second" class="ui-btn-right">Next</a>
    </div>

    <div data-role="content">
        <a href="#" data-role="button" id="test-button">Test button</a>
    </div>

    <div data-theme="a" data-role="footer" data-position="fixed">

    </div>
</div>

To execute code that will only available to the index page we could use this syntax:

$('#index').on('pageinit', function() {

});

Pageinit event will be executed every time page is about be be loaded and shown for the first time. It will not trigger again unless page is manually refreshed or Ajax page loading is turned off. In case you want code to execute every time you visit a page it is better to use pagebeforeshow event.

Here's a working example: http://jsfiddle.net/Gajotres/Q3Usv/ to demonstrate this problem.

Few more notes on this question. No matter if you are using 1 html multiple pages or multiple HTML files paradigm it is advised to separate all of your custom JavaScript page handling into a single separate JavaScript file. This will note make your code any better but you will have much better code overview, especially while creating a jQuery Mobile application.

There's also another special jQuery Mobile event and it is called mobileinit. When jQuery Mobile starts, it triggers a mobileinit event on the document object. To override default settings, bind them to mobileinit. One of a good examples of mobileinit usage is turning off Ajax page loading, or changing default Ajax loader behavior.

$(document).on("mobileinit", function(){
  //apply overrides here
});

Page events transition order

First all events can be found here: http://api.jquerymobile.com/category/events/

Lets say we have a page A and a page B, this is a unload/load order:

  1. page B - event pagebeforecreate

  2. page B - event pagecreate

  3. page B - event pageinit

  4. page A - event pagebeforehide

  5. page A - event pageremove

  6. page A - event pagehide

  7. page B - event pagebeforeshow

  8. page B - event pageshow

For better page events understanding read this:

  • pagebeforeload, pageload and pageloadfailed are fired when an external page is loaded
  • pagebeforechange, pagechange and pagechangefailed are page change events. These events are fired when a user is navigating between pages in the applications.
  • pagebeforeshow, pagebeforehide, pageshow and pagehide are page transition events. These events are fired before, during and after a transition and are named.
  • pagebeforecreate, pagecreate and pageinit are for page initialization.
  • pageremove can be fired and then handled when a page is removed from the DOM

Page loading jsFiddle example: http://jsfiddle.net/Gajotres/QGnft/

If AJAX is not enabled, some events may not fire.

Prevent page transition

If for some reason page transition needs to be prevented on some condition it can be done with this code:

$(document).on('pagebeforechange', function(e, data){
    var to = data.toPage,
        from = data.options.fromPage;

    if (typeof to  === 'string') {
        var u = $.mobile.path.parseUrl(to);
        to = u.hash || '#' + u.pathname.substring(1);
        if (from) from = '#' + from.attr('id');

        if (from === '#index' && to === '#second') {
            alert('Can not transition from #index to #second!');
            e.preventDefault();
            e.stopPropagation();

            // remove active status on a button, if transition was triggered with a button
            $.mobile.activePage.find('.ui-btn-active').removeClass('ui-btn-active ui-focus ui-btn');;
        }
    }
});

This example will work in any case because it will trigger at a begging of every page transition and what is most important it will prevent page change before page transition can occur.

Here's a working example:

Prevent multiple event binding/triggering

jQuery Mobile works in a different way than classic web applications. Depending on how you managed to bind your events each time you visit some page it will bind events over and over. This is not an error, it is simply how jQuery Mobile handles its pages. For example, take a look at this code snippet:

$(document).on('pagebeforeshow','#index' ,function(e,data){
    $(document).on('click', '#test-button',function(e) {
        alert('Button click');
    });
});

Working jsFiddle example: http://jsfiddle.net/Gajotres/CCfL4/

Each time you visit page #index click event will is going to be bound to button #test-button. Test it by moving from page 1 to page 2 and back several times. There are few ways to prevent this problem:

Solution 1

Best solution would be to use pageinit to bind events. If you take a look at an official documentation you will find out that pageinit will trigger ONLY once, just like document ready, so there's no way events will be bound again. This is best solution because you don't have processing overhead like when removing events with off method.

Working jsFiddle example: http://jsfiddle.net/Gajotres/AAFH8/

This working solution is made on a basis of a previous problematic example.

Solution 2

Remove event before you bind it:

$(document).on('pagebeforeshow', '#index', function(){
    $(document).off('click', '#test-button').on('click', '#test-button',function(e) {
        alert('Button click');
    });
});

Working jsFiddle example: http://jsfiddle.net/Gajotres/K8YmG/

Solution 3

Use a jQuery Filter selector, like this:

$('#carousel div:Event(!click)').each(function(){
    //If click is not bind to #carousel div do something
});

Because event filter is not a part of official jQuery framework it can be found here: http://www.codenothing.com/archives/2009/event-filter/

In a nutshell, if speed is your main concern then Solution 2 is much better than Solution 1.

Solution 4

A new one, probably an easiest of them all.

$(document).on('pagebeforeshow', '#index', function(){
    $(document).on('click', '#test-button',function(e) {
        if(e.handled !== true) // This will prevent event triggering more than once
        {
            alert('Clicked');
            e.handled = true;
        }
    });
});

Working jsFiddle example: http://jsfiddle.net/Gajotres/Yerv9/

Tnx to the sholsinger for this solution: http://sholsinger.com/archive/2011/08/prevent-jquery-live-handlers-from-firing-multiple-times/

pageChange event quirks - triggering twice

Sometimes pagechange event can trigger twice and it does not have anything to do with the problem mentioned before.

The reason the pagebeforechange event occurs twice is due to the recursive call in changePage when toPage is not a jQuery enhanced DOM object. This recursion is dangerous, as the developer is allowed to change the toPage within the event. If the developer consistently sets toPage to a string, within the pagebeforechange event handler, regardless of whether or not it was an object an infinite recursive loop will result. The pageload event passes the new page as the page property of the data object (This should be added to the documentation, it's not listed currently). The pageload event could therefore be used to access the loaded page.

In few words this is happening because you are sending additional parameters through pageChange.

Example:

<a data-role="button" data-icon="arrow-r" data-iconpos="right" href="#care-plan-view?id=9e273f31-2672-47fd-9baa-6c35f093a800&amp;name=Sat"><h3>Sat</h3></a>

To fix this problem use any page event listed in Page events transition order.

Page Change Times

As mentioned, when you change from one jQuery Mobile page to another, typically either through clicking on a link to another jQuery Mobile page that already exists in the DOM, or by manually calling $.mobile.changePage, several events and subsequent actions occur. At a high level the following actions occur:

  • A page change process is begun
  • A new page is loaded
  • The content for that page is “enhanced” (styled)
  • A transition (slide/pop/etc) from the existing page to the new page occurs

This is a average page transition benchmark:

Page load and processing: 3 ms

Page enhance: 45 ms

Transition: 604 ms

Total time: 670 ms

*These values are in milliseconds.

So as you can see a transition event is eating almost 90% of execution time.

Data/Parameters manipulation between page transitions

It is possible to send a parameter/s from one page to another during page transition. It can be done in few ways.

Reference: https://stackoverflow.com/a/13932240/1848600

Solution 1:

You can pass values with changePage:

$.mobile.changePage('page2.html', { dataUrl : "page2.html?paremeter=123", data : { 'paremeter' : '123' }, reloadPage : true, changeHash : true });

And read them like this:

$(document).on('pagebeforeshow', "#index", function (event, data) {
    var parameters = $(this).data("url").split("?")[1];;
    parameter = parameters.replace("parameter=","");
    alert(parameter);
});

Example:

index.html

_x000D_
_x000D_
<!DOCTYPE html>_x000D_
  <html>_x000D_
    <head>_x000D_
    <meta charset="utf-8" />_x000D_
    <meta name="viewport" content="widdiv=device-widdiv, initial-scale=1.0, maximum-scale=1.0, user-scalable=no" />_x000D_
    <meta name="apple-mobile-web-app-capable" content="yes" />_x000D_
    <meta name="apple-mobile-web-app-status-bar-style" content="black" />_x000D_
    <title>_x000D_
    </title>_x000D_
    <link rel="stylesheet" href="http://code.jquery.com/mobile/1.2.0/jquery.mobile-1.2.0.min.css" />_x000D_
    <script src="http://www.dragan-gaic.info/js/jquery-1.8.2.min.js">_x000D_
    </script>_x000D_
    <script src="http://code.jquery.com/mobile/1.2.0/jquery.mobile-1.2.0.min.js"></script>_x000D_
    <script>_x000D_
        $(document).on('pagebeforeshow', "#index",function () {_x000D_
            $(document).on('click', "#changePage",function () {_x000D_
                $.mobile.changePage('second.html', { dataUrl : "second.html?paremeter=123", data : { 'paremeter' : '123' }, reloadPage : false, changeHash : true });_x000D_
            });_x000D_
        });_x000D_
_x000D_
        $(document).on('pagebeforeshow', "#second",function () {_x000D_
            var parameters = $(this).data("url").split("?")[1];;_x000D_
            parameter = parameters.replace("parameter=","");_x000D_
            alert(parameter);_x000D_
        });_x000D_
    </script>_x000D_
   </head>_x000D_
   <body>_x000D_
    <!-- Home -->_x000D_
    <div data-role="page" id="index">_x000D_
        <div data-role="header">_x000D_
            <h3>_x000D_
                First Page_x000D_
            </h3>_x000D_
        </div>_x000D_
        <div data-role="content">_x000D_
          <a data-role="button" id="changePage">Test</a>_x000D_
        </div> <!--content-->_x000D_
    </div><!--page-->_x000D_
_x000D_
  </body>_x000D_
</html>
_x000D_
_x000D_
_x000D_

second.html

_x000D_
_x000D_
<!DOCTYPE html>_x000D_
  <html>_x000D_
    <head>_x000D_
    <meta charset="utf-8" />_x000D_
    <meta name="viewport" content="widdiv=device-widdiv, initial-scale=1.0, maximum-scale=1.0, user-scalable=no" />_x000D_
    <meta name="apple-mobile-web-app-capable" content="yes" />_x000D_
    <meta name="apple-mobile-web-app-status-bar-style" content="black" />_x000D_
    <title>_x000D_
    </title>_x000D_
    <link rel="stylesheet" href="http://code.jquery.com/mobile/1.2.0/jquery.mobile-1.2.0.min.css" />_x000D_
    <script src="http://www.dragan-gaic.info/js/jquery-1.8.2.min.js">_x000D_
    </script>_x000D_
    <script src="http://code.jquery.com/mobile/1.2.0/jquery.mobile-1.2.0.min.js"></script>_x000D_
   </head>_x000D_
   <body>_x000D_
    <!-- Home -->_x000D_
    <div data-role="page" id="second">_x000D_
        <div data-role="header">_x000D_
            <h3>_x000D_
                Second Page_x000D_
            </h3>_x000D_
        </div>_x000D_
        <div data-role="content">_x000D_
_x000D_
        </div> <!--content-->_x000D_
    </div><!--page-->_x000D_
_x000D_
  </body>_x000D_
</html>
_x000D_
_x000D_
_x000D_

Solution 2:

Or you can create a persistent JavaScript object for a storage purpose. As long Ajax is used for page loading (and page is not reloaded in any way) that object will stay active.

var storeObject = {
    firstname : '',
    lastname : ''
}

Example: http://jsfiddle.net/Gajotres/9KKbx/

Solution 3:

You can also access data from the previous page like this:

$(document).on('pagebeforeshow', '#index',function (e, data) {
    alert(data.prevPage.attr('id'));
});

prevPage object holds a complete previous page.

Solution 4:

As a last solution we have a nifty HTML implementation of localStorage. It only works with HTML5 browsers (including Android and iOS browsers) but all stored data is persistent through page refresh.

if(typeof(Storage)!=="undefined") {
    localStorage.firstname="Dragan";
    localStorage.lastname="Gaic";
}

Example: http://jsfiddle.net/Gajotres/J9NTr/

Probably best solution but it will fail in some versions of iOS 5.X. It is a well know error.

Don’t Use .live() / .bind() / .delegate()

I forgot to mention (and tnx andleer for reminding me) use on/off for event binding/unbinding, live/die and bind/unbind are deprecated.

The .live() method of jQuery was seen as a godsend when it was introduced to the API in version 1.3. In a typical jQuery app there can be a lot of DOM manipulation and it can become very tedious to hook and unhook as elements come and go. The .live() method made it possible to hook an event for the life of the app based on its selector. Great right? Wrong, the .live() method is extremely slow. The .live() method actually hooks its events to the document object, which means that the event must bubble up from the element that generated the event until it reaches the document. This can be amazingly time consuming.

It is now deprecated. The folks on the jQuery team no longer recommend its use and neither do I. Even though it can be tedious to hook and unhook events, your code will be much faster without the .live() method than with it.

Instead of .live() you should use .on(). .on() is about 2-3x faster than .live(). Take a look at this event binding benchmark: http://jsperf.com/jquery-live-vs-delegate-vs-on/34, everything will be clear from there.

Benchmarking:

There's an excellent script made for jQuery Mobile page events benchmarking. It can be found here: https://github.com/jquery/jquery-mobile/blob/master/tools/page-change-time.js. But before you do anything with it I advise you to remove its alert notification system (each “change page” is going to show you this data by halting the app) and change it to console.log function.

Basically this script will log all your page events and if you read this article carefully (page events descriptions) you will know how much time jQm spent of page enhancements, page transitions ....

Final notes

Always, and I mean always read official jQuery Mobile documentation. It will usually provide you with needed information, and unlike some other documentation this one is rather good, with enough explanations and code examples.

Changes:

  • 30.01.2013 - Added a new method of multiple event triggering prevention
  • 31.01.2013 - Added a better clarification for chapter Data/Parameters manipulation between page transitions
  • 03.02.2013 - Added new content/examples to the chapter Data/Parameters manipulation between page transitions
  • 22.05.2013 - Added a solution for page transition/change prevention and added links to the official page events API documentation
  • 18.05.2013 - Added another solution against multiple event binding

How to grant "grant create session" privilege?

You would use the WITH ADMIN OPTION option in the GRANT statement

GRANT CREATE SESSION TO <<username>> WITH ADMIN OPTION

.bashrc: Permission denied

If you want to edit that file (or any file in generally), you can't edit it simply writing its name in terminal. You must to use a command to a text editor to do this. For example:

nano ~/.bashrc

or

gedit ~/.bashrc

And in general, for any type of file:

xdg-open ~/.bashrc

Writing only ~/.bashrc in terminal, this will try to execute that file, but .bashrc file is not meant to be an executable file. If you want to execute the code inside of it, you can source it like follow:

source ~/.bashrc

or simple:

. ~/.bashrc 

Javascript: open new page in same window

Here's what worked for me:

<button name="redirect" onClick="redirect()">button name</button>
<script type="text/javascript">
function redirect(){
var url = "http://www.google.com";
window.open(url, '_top');
}
</script>

pass JSON to HTTP POST Request

You don't want multipart, but a "plain" POST request (with Content-Type: application/json) instead. Here is all you need:

var request = require('request');

var requestData = {
  request: {
    slice: [
      {
        origin: "ZRH",
        destination: "DUS",
        date: "2014-12-02"
      }
    ],
    passengers: {
      adultCount: 1,
      infantInLapCount: 0,
      infantInSeatCount: 0,
      childCount: 0,
      seniorCount: 0
    },
    solutions: 2,
    refundable: false
  }
};

request('https://www.googleapis.com/qpxExpress/v1/trips/search?key=myApiKey',
        { json: true, body: requestData },
        function(err, res, body) {
  // `body` is a js object if request was successful
});

mongo - couldn't connect to server 127.0.0.1:27017

Another solution that resolved this same error for me, though this one might only be if you are accessing mongo (locally) via a SSH connection via a virtual machine (at least that's my setup), and may be a linux specific environment variable issue:

export LC_ALL=C

Also, I have had more success running mongo as a daemon service then with sudo service start, my understanding is this uses command line arguments rather then the config file so it likely is an issue with my config file:

mongod --fork --logpath /var/log/mongodb.log --auth --port 27017 --dbpath /var/lib/mongodb/admin

How do I get the number of elements in a list?

To get the number of element in any iterable object, your goto method in Python is len() eg.

a = range(1000) # range
b = 'abcdefghijklmnopqrstuvwxyz' # string
c = [10, 20, 30] # List
d = (30, 40, 50, 60, 70) # tuple
e = {11, 21, 31, 41} # set

len() method can work on all the above data types because they are iterable i.e You can iterate over them.

all_var = [a, b, c, d, e] # All variables are stored to a list
for var in all_var:
    print(len(var))

Rough estimate of the len() method

def len(iterable, /):
    total = 0
    for i in iterable:
        total += 1
    return total

How can I measure the actual memory usage of an application or process?

I would suggest that you use atop. You can find everything about it on this page. It is capable of providing all the necessary KPI for your processes and it can also capture to a file.

Assign variable value inside if-statement

You can assign, but not declare, inside an if:

Try this:

int v; // separate declaration
if((v = someMethod()) != 0) return true;

I/O error(socket error): [Errno 111] Connection refused

Its seems that server is not running properly so ensure that with terminal by

telnet ip port

example

telnet localhost 8069

It will return connected to localhost so it indicates that there is no problem with the connection Else it will return Connection refused it indicates that there is problem with the connection

Include another HTML file in a HTML file

You can do that with JavaScript's library jQuery like this:

HTML:

<div class="banner" title="banner.html"></div>

JS:

$(".banner").each(function(){
    var inc=$(this);
    $.get(inc.attr("title"), function(data){
        inc.replaceWith(data);
    });
});

Please note that banner.html should be located under the same domain your other pages are in otherwise your webpages will refuse the banner.html file due to Cross-Origin Resource Sharing policies.

Also, please note that if you load your content with JavaScript, Google will not be able to index it so it's not exactly a good method for SEO reasons.

How to redirect output of systemd service to a file

Short answer:

StandardOutput=file:/var/log1.log
StandardError=file:/var/log2.log

If you don't want the files to be cleared every time the service is run, use append instead:

StandardOutput=append:/var/log1.log
StandardError=append:/var/log2.log

SQLite string contains other string query

While LIKE is suitable for this case, a more general purpose solution is to use instr, which doesn't require characters in the search string to be escaped. Note: instr is available starting from Sqlite 3.7.15.

SELECT *
  FROM TABLE  
 WHERE instr(column, 'cats') > 0;

Also, keep in mind that LIKE is case-insensitive, whereas instr is case-sensitive.

Python: One Try Multiple Except

Yes, it is possible.

try:
   ...
except FirstException:
   handle_first_one()

except SecondException:
   handle_second_one()

except (ThirdException, FourthException, FifthException) as e:
   handle_either_of_3rd_4th_or_5th()

except Exception:
   handle_all_other_exceptions()

See: http://docs.python.org/tutorial/errors.html

The "as" keyword is used to assign the error to a variable so that the error can be investigated more thoroughly later on in the code. Also note that the parentheses for the triple exception case are needed in python 3. This page has more info: Catch multiple exceptions in one line (except block)

Pass entire form as data in jQuery Ajax function

There's a function that does exactly this:

http://api.jquery.com/serialize/

var data = $('form').serialize();
$.post('url', data);

The term "Add-Migration" is not recognized

I had this problem and none of the previous solutions helped me. My problem was actually due to an outdated version of powershell on my Windows 7 machine - once I updated to powershell 5 it started working.

Clip/Crop background-image with CSS

may be you can write like this:

#graphic { 
 background-image: url(image.jpg); 
 background-position: 0 -50px; 
 width: 200px; 
 height: 100px;
}

Modal width (increase)

Bootstrap 4 includes sizing utilities. You can change the size to 25/50/75/100% of the page width (I wish there were even more increments).

To use these we will replace the modal-lg class. Both the default width and modal-lg use css max-width to control the modal's width, so first add the mw-100 class to effectively disable max-width. Then just add the width class you want, e.g. w-75.

Note that you should place the mw-100 and w-75 classes in the div with the modal-dialog class, not the modal div e.g.,

<div class='modal-dialog mw-100 w-75'>
    ...
</div>

exception in initializer error in java when using Netbeans

I got same error and it was due to older Lombok version. Check and update your Lombok version, Changes in Lombok

v1.18.4 - Many improvements for lombok's JDK10/11 support.

How to move table from one tablespace to another in oracle 11g

Try this to move your table (tbl1) to tablespace (tblspc2).

alter table tb11 move tablespace tblspc2;

How to write dynamic variable in Ansible playbook

my_var: the variable declared

VAR: the variable, whose value is to be checked

param_1, param_2: values of the variable VAR

value_1, value_2, value_3: the values to be assigned to my_var according to the values of my_var

my_var: "{{ 'value_1' if VAR == 'param_1' else 'value_2' if VAR == 'param_2' else 'value_3' }}"

Adding two Java 8 streams, or an extra element to a stream

You can use Guava's Streams.concat(Stream<? extends T>... streams) method, which will be very short with static imports:

Stream stream = concat(stream1, stream2, of(element));

Can a JSON value contain a multiline string

Per the specification, the JSON grammar's char production can take the following values:

  • any-Unicode-character-except-"-or-\-or-control-character
  • \"
  • \\
  • \/
  • \b
  • \f
  • \n
  • \r
  • \t
  • \u four-hex-digits

Newlines are "control characters", so no, you may not have a literal newline within your string. However, you may encode it using whatever combination of \n and \r you require.

The JSONLint tool confirms that your JSON is invalid.


And, if you want to write newlines inside your JSON syntax without actually including newlines in the data, then you're doubly out of luck. While JSON is intended to be human-friendly to a degree, it is still data and you're trying to apply arbitrary formatting to that data. That is absolutely not what JSON is about.

How to find distinct rows with field in list using JPA and Spring?

I finally was able to figure out a simple solution without the @Query annotation.

List<People> findDistinctByNameNotIn(List<String> names);

Of course, I got the people object instead of only Strings. I can then do the change in java.

When do we need curly braces around shell variables?

In this particular example, it makes no difference. However, the {} in ${} are useful if you want to expand the variable foo in the string

"${foo}bar"

since "$foobar" would instead expand the variable identified by foobar.

Curly braces are also unconditionally required when:

  • expanding array elements, as in ${array[42]}
  • using parameter expansion operations, as in ${filename%.*} (remove extension)
  • expanding positional parameters beyond 9: "$8 $9 ${10} ${11}"

Doing this everywhere, instead of just in potentially ambiguous cases, can be considered good programming practice. This is both for consistency and to avoid surprises like $foo_$bar.jpg, where it's not visually obvious that the underscore becomes part of the variable name.

sql primary key and index

Here the passage from the MSDN:

When you specify a PRIMARY KEY constraint for a table, the Database Engine enforces data uniqueness by creating a unique index for the primary key columns. This index also permits fast access to data when the primary key is used in queries. Therefore, the primary keys that are chosen must follow the rules for creating unique indexes.

ExecuteReader requires an open and available Connection. The connection's current state is Connecting

Sorry for only commenting in the first place, but i'm posting almost every day a similar comment since many people think that it would be smart to encapsulate ADO.NET functionality into a DB-Class(me too 10 years ago). Mostly they decide to use static/shared objects since it seems to be faster than to create a new object for any action.

That is neither a good idea in terms of peformance nor in terms of fail-safety.

Don't poach on the Connection-Pool's territory

There's a good reason why ADO.NET internally manages the underlying Connections to the DBMS in the ADO-NET Connection-Pool:

In practice, most applications use only one or a few different configurations for connections. This means that during application execution, many identical connections will be repeatedly opened and closed. To minimize the cost of opening connections, ADO.NET uses an optimization technique called connection pooling.

Connection pooling reduces the number of times that new connections must be opened. The pooler maintains ownership of the physical connection. It manages connections by keeping alive a set of active connections for each given connection configuration. Whenever a user calls Open on a connection, the pooler looks for an available connection in the pool. If a pooled connection is available, it returns it to the caller instead of opening a new connection. When the application calls Close on the connection, the pooler returns it to the pooled set of active connections instead of closing it. Once the connection is returned to the pool, it is ready to be reused on the next Open call.

So obviously there's no reason to avoid creating,opening or closing connections since actually they aren't created,opened and closed at all. This is "only" a flag for the connection pool to know when a connection can be reused or not. But it's a very important flag, because if a connection is "in use"(the connection pool assumes), a new physical connection must be openend to the DBMS what is very expensive.

So you're gaining no performance improvement but the opposite. If the maximum pool size specified (100 is the default) is reached, you would even get exceptions(too many open connections ...). So this will not only impact the performance tremendously but also be a source for nasty errors and (without using Transactions) a data-dumping-area.

If you're even using static connections you're creating a lock for every thread trying to access this object. ASP.NET is a multithreading environment by nature. So theres a great chance for these locks which causes performance issues at best. Actually sooner or later you'll get many different exceptions(like your ExecuteReader requires an open and available Connection).

Conclusion:

  • Don't reuse connections or any ADO.NET objects at all.
  • Don't make them static/shared(in VB.NET)
  • Always create, open(in case of Connections), use, close and dispose them where you need them(f.e. in a method)
  • use the using-statement to dispose and close(in case of Connections) implicitely

That's true not only for Connections(although most noticable). Every object implementing IDisposable should be disposed(simplest by using-statement), all the more in the System.Data.SqlClient namespace.

All the above speaks against a custom DB-Class which encapsulates and reuse all objects. That's the reason why i commented to trash it. That's only a problem source.


Edit: Here's a possible implementation of your retrievePromotion-method:

public Promotion retrievePromotion(int promotionID)
{
    Promotion promo = null;
    var connectionString = System.Configuration.ConfigurationManager.ConnectionStrings["MainConnStr"].ConnectionString;
    using (SqlConnection connection = new SqlConnection(connectionString))
    {
        var queryString = "SELECT PromotionID, PromotionTitle, PromotionURL FROM Promotion WHERE PromotionID=@PromotionID";
        using (var da = new SqlDataAdapter(queryString, connection))
        {
            // you could also use a SqlDataReader instead
            // note that a DataTable does not need to be disposed since it does not implement IDisposable
            var tblPromotion = new DataTable();
            // avoid SQL-Injection
            da.SelectCommand.Parameters.Add("@PromotionID", SqlDbType.Int);
            da.SelectCommand.Parameters["@PromotionID"].Value = promotionID;
            try
            {
                connection.Open(); // not necessarily needed in this case because DataAdapter.Fill does it otherwise 
                da.Fill(tblPromotion);
                if (tblPromotion.Rows.Count != 0)
                {
                    var promoRow = tblPromotion.Rows[0];
                    promo = new Promotion()
                    {
                        promotionID    = promotionID,
                        promotionTitle = promoRow.Field<String>("PromotionTitle"),
                        promotionUrl   = promoRow.Field<String>("PromotionURL")
                    };
                }
            }
            catch (Exception ex)
            {
                // log this exception or throw it up the StackTrace
                // we do not need a finally-block to close the connection since it will be closed implicitely in an using-statement
                throw;
            }
        }
    }
    return promo;
}

What is the difference between resource and endpoint?

Consider a server which has the information of users, missions and their reward points.

  1. Users and Reward Points are the resources
  2. An end point can relate to more than one resource
  3. Endpoints can be described using either a description or a full or partial URL

enter image description here

Source: API Endpoints vs Resources

When and where to use GetType() or typeof()?

You may find it easier to use the is keyword:

if (mycontrol is TextBox)

What's the difference between struct and class in .NET?

Structure vs Class

A structure is a value type so it is stored on the stack, but a class is a reference type and is stored on the heap.

A structure doesn't support inheritance, and polymorphism, but a class supports both.

By default, all the struct members are public but class members are by default private in nature.

As a structure is a value type, we can't assign null to a struct object, but it is not the case for a class.

Check if a String is in an ArrayList of Strings

    List list1 = new ArrayList();
    list1.add("one");
    list1.add("three");
    list1.add("four");

    List list2 = new ArrayList();
    list2.add("one");
    list2.add("two");
    list2.add("three");
    list2.add("four");
    list2.add("five");


    list2.stream().filter( x -> !list1.contains(x) ).forEach(x -> System.out.println(x));

The output is:

two
five

What is the difference between the float and integer data type when the size is the same?

  • float stores floating-point values, that is, values that have potential decimal places
  • int only stores integral values, that is, whole numbers

So while both are 32 bits wide, their use (and representation) is quite different. You cannot store 3.141 in an integer, but you can in a float.

Dissecting them both a little further:

In an integer, all bits are used to store the number value. This is (in Java and many computers too) done in the so-called two's complement. This basically means that you can represent the values of −231 to 231 − 1.

In a float, those 32 bits are divided between three distinct parts: The sign bit, the exponent and the mantissa. They are laid out as follows:

S EEEEEEEE MMMMMMMMMMMMMMMMMMMMMMM

There is a single bit that determines whether the number is negative or non-negative (zero is neither positive nor negative, but has the sign bit set to zero). Then there are eight bits of an exponent and 23 bits of mantissa. To get a useful number from that, (roughly) the following calculation is performed:

M × 2E

(There is more to it, but this should suffice for the purpose of this discussion)

The mantissa is in essence not much more than a 24-bit integer number. This gets multiplied by 2 to the power of the exponent part, which, roughly, is a number between −128 and 127.

Therefore you can accurately represent all numbers that would fit in a 24-bit integer but the numeric range is also much greater as larger exponents allow for larger values. For example, the maximum value for a float is around 3.4 × 1038 whereas int only allows values up to 2.1 × 109.

But that also means, since 32 bits only have 4.2 × 109 different states (which are all used to represent the values int can store), that at the larger end of float's numeric range the numbers are spaced wider apart (since there cannot be more unique float numbers than there are unique int numbers). You cannot represent some numbers exactly, then. For example, the number 2 × 1012 has a representation in float of 1,999,999,991,808. That might be close to 2,000,000,000,000 but it's not exact. Likewise, adding 1 to that number does not change it because 1 is too small to make a difference in the larger scales float is using there.

Similarly, you can also represent very small numbers (between 0 and 1) in a float but regardless of whether the numbers are very large or very small, float only has a precision of around 6 or 7 decimal digits. If you have large numbers those digits are at the start of the number (e.g. 4.51534 × 1035, which is nothing more than 451534 follows by 30 zeroes – and float cannot tell anything useful about whether those 30 digits are actually zeroes or something else), for very small numbers (e.g. 3.14159 × 10−27) they are at the far end of the number, way beyond the starting digits of 0.0000...

Replace spaces with dashes and make all letters lower-case

Just use the String replace and toLowerCase methods, for example:

var str = "Sonic Free Games";
str = str.replace(/\s+/g, '-').toLowerCase();
console.log(str); // "sonic-free-games"

Notice the g flag on the RegExp, it will make the replacement globally within the string, if it's not used, only the first occurrence will be replaced, and also, that RegExp will match one or more white-space characters.

How can I update a single row in a ListView?

int wantedPosition = 25; // Whatever position you're looking for
int firstPosition = linearLayoutManager.findFirstVisibleItemPosition(); // This is the same as child #0
int wantedChild = wantedPosition - firstPosition;

if (wantedChild < 0 || wantedChild >= linearLayoutManager.getChildCount()) {
    Log.w(TAG, "Unable to get view for desired position, because it's not being displayed on screen.");
    return;
}

View wantedView = linearLayoutManager.getChildAt(wantedChild);
mlayoutOver =(LinearLayout)wantedView.findViewById(R.id.layout_over);
mlayoutPopup = (LinearLayout)wantedView.findViewById(R.id.layout_popup);

mlayoutOver.setVisibility(View.INVISIBLE);
mlayoutPopup.setVisibility(View.VISIBLE);

For RecycleView please use this code

How to make a rest post call from ReactJS code?

I think this way also a normal way. But sorry, I can't describe in English ((

_x000D_
_x000D_
    submitHandler = e => {_x000D_
    e.preventDefault()_x000D_
    console.log(this.state)_x000D_
    fetch('http://localhost:5000/questions',{_x000D_
        method: 'POST',_x000D_
        headers: {_x000D_
            Accept: 'application/json',_x000D_
                    'Content-Type': 'application/json',_x000D_
        },_x000D_
        body: JSON.stringify(this.state)_x000D_
    }).then(response => {_x000D_
            console.log(response)_x000D_
        })_x000D_
        .catch(error =>{_x000D_
            console.log(error)_x000D_
        })_x000D_
    _x000D_
}
_x000D_
_x000D_
_x000D_

https://googlechrome.github.io/samples/fetch-api/fetch-post.html

fetch('url/questions',{ method: 'POST', headers: { Accept: 'application/json', 'Content-Type': 'application/json', }, body: JSON.stringify(this.state) }).then(response => { console.log(response) }) .catch(error =>{ console.log(error) })

Linux shell script for database backup

#!/bin/bash

# Add your backup dir location, password, mysql location and mysqldump        location
DATE=$(date +%d-%m-%Y)
BACKUP_DIR="/var/www/back"
MYSQL_USER="root"
MYSQL_PASSWORD=""
MYSQL='/usr/bin/mysql'
MYSQLDUMP='/usr/bin/mysqldump'
DB='demo'

#to empty the backup directory and delete all previous backups
rm -r $BACKUP_DIR/*  

mysqldump -u root -p'' demo | gzip -9 > $BACKUP_DIR/demo$date_format.sql.$DATE.gz

#changing permissions of directory 
chmod -R 777 $BACKUP_DIR

Can you nest html forms?

Use empty form tag before your nested form

Tested and Worked on Firefox, Chrome

Not Tested on I.E.

<form name="mainForm" action="mainAction">
  <form></form>
  <form name="subForm"  action="subAction">
  </form>
</form>

EDIT by @adusza: As the commenters pointed out, the above code does not result in nested forms. However, if you add div elements like below, you will have subForm inside mainForm, and the first blank form will be removed.

<form name="mainForm" action="mainAction">
  <div>
      <form></form>
      <form name="subForm"  action="subAction">
      </form>
  </div>
</form>

What is the functionality of setSoTimeout and how it works?

The JavaDoc explains it very well:

With this option set to a non-zero timeout, a read() call on the InputStream associated with this Socket will block for only this amount of time. If the timeout expires, a java.net.SocketTimeoutException is raised, though the Socket is still valid. The option must be enabled prior to entering the blocking operation to have effect. The timeout must be > 0. A timeout of zero is interpreted as an infinite timeout.

SO_TIMEOUT is the timeout that a read() call will block. If the timeout is reached, a java.net.SocketTimeoutException will be thrown. If you want to block forever put this option to zero (the default value), then the read() call will block until at least 1 byte could be read.

How to change title of Activity in Android?

@Override
protected void onCreate(Bundle savedInstanceState) {
    super.onCreate(savedInstanceState);
    setContentView(R.layout.Main_Activity);
    this.setTitle("Title name");
}

Unable to call the built in mb_internal_encoding method?

If someone is having trouble with installing php-mbstring package in ubuntu do following sudo apt-get install libapache2-mod-php5

REST / SOAP endpoints for a WCF service

This is what i did to make it work. Make sure you put
webHttp automaticFormatSelectionEnabled="true" inside endpoint behaviour.

[ServiceContract]
public interface ITestService
{

    [WebGet(BodyStyle = WebMessageBodyStyle.Bare, UriTemplate = "/product", ResponseFormat = WebMessageFormat.Json)]
    string GetData();
}

public class TestService : ITestService
{
    public string GetJsonData()
    {
        return "I am good...";
    }
}

Inside service model

   <service name="TechCity.Business.TestService">

    <endpoint address="soap" binding="basicHttpBinding" name="SoapTest"
      bindingName="BasicSoap" contract="TechCity.Interfaces.ITestService" />
    <endpoint address="mex"
              contract="IMetadataExchange" binding="mexHttpBinding"/>
    <endpoint behaviorConfiguration="jsonBehavior" binding="webHttpBinding"
              name="Http" contract="TechCity.Interfaces.ITestService" />
    <host>
      <baseAddresses>
        <add baseAddress="http://localhost:8739/test" />
      </baseAddresses>
    </host>
  </service>

EndPoint Behaviour

  <endpointBehaviors>
    <behavior name="jsonBehavior">
      <webHttp automaticFormatSelectionEnabled="true"  />
      <!-- use JSON serialization -->
    </behavior>
  </endpointBehaviors>

multiple prints on the same line in Python

print() has a built in parameter "end" that is by default set to "\n" Calling print("This is America") is actually calling print("This is America", end = "\n"). An easy way to do is to call print("This is America", end ="")

Check if boolean is true?

Both are correct.

You probably have some coding standard in your company - just see to follow it through. If you don't have - you should :)

Echo newline in Bash prints literal \n

Sometimes you can pass multiple strings separated by a space and it will be interpreted as \n.

For example when using a shell script for multi-line notifcations:

#!/bin/bash
notify-send 'notification success' 'another line' 'time now '`date +"%s"`

unable to install pg gem

If you are using jruby instead of ruby you will have similar issues when installing the pg gem. Instead you need to install the adaptor:

gem 'activerecord-jdbcpostgresql-adapter'

How do I set default value of select box in angularjs

After searching and trying multiple non working options to get my select default option working. I find a clean solution at: http://www.undefinednull.com/2014/08/11/a-brief-walk-through-of-the-ng-options-in-angularjs/

<select class="ajg-stereo-fader-input-name ajg-select-left"  ng-options="option.name for option in selectOptions" ng-model="inputLeft"></select>
<select class="ajg-stereo-fader-input-name ajg-select-right" ng-options="option.name for option in selectOptions" ng-model="inputRight"></select>

 scope.inputLeft =  scope.selectOptions[0];
 scope.inputRight = scope.selectOptions[1];

How can I make one python file run another?

You could use this script:

def run(runfile):
  with open(runfile,"r") as rnf:
    exec(rnf.read())

Syntax:

run("file.py")

Algorithm to return all combinations of k elements from n

Here is my proposition in C++

I tried to impose as little restriction on the iterator type as i could so this solution assumes just forward iterator, and it can be a const_iterator. This should work with any standard container. In cases where arguments don't make sense it throws std::invalid_argumnent

#include <vector>
#include <stdexcept>

template <typename Fci> // Fci - forward const iterator
std::vector<std::vector<Fci> >
enumerate_combinations(Fci begin, Fci end, unsigned int combination_size)
{
    if(begin == end && combination_size > 0u)
        throw std::invalid_argument("empty set and positive combination size!");
    std::vector<std::vector<Fci> > result; // empty set of combinations
    if(combination_size == 0u) return result; // there is exactly one combination of
                                              // size 0 - emty set
    std::vector<Fci> current_combination;
    current_combination.reserve(combination_size + 1u); // I reserve one aditional slot
                                                        // in my vector to store
                                                        // the end sentinel there.
                                                        // The code is cleaner thanks to that
    for(unsigned int i = 0u; i < combination_size && begin != end; ++i, ++begin)
    {
        current_combination.push_back(begin); // Construction of the first combination
    }
    // Since I assume the itarators support only incrementing, I have to iterate over
    // the set to get its size, which is expensive. Here I had to itrate anyway to  
    // produce the first cobination, so I use the loop to also check the size.
    if(current_combination.size() < combination_size)
        throw std::invalid_argument("combination size > set size!");
    result.push_back(current_combination); // Store the first combination in the results set
    current_combination.push_back(end); // Here I add mentioned earlier sentinel to
                                        // simplyfy rest of the code. If I did it 
                                        // earlier, previous statement would get ugly.
    while(true)
    {
        unsigned int i = combination_size;
        Fci tmp;                            // Thanks to the sentinel I can find first
        do                                  // iterator to change, simply by scaning
        {                                   // from right to left and looking for the
            tmp = current_combination[--i]; // first "bubble". The fact, that it's 
            ++tmp;                          // a forward iterator makes it ugly but I
        }                                   // can't help it.
        while(i > 0u && tmp == current_combination[i + 1u]);

        // Here is probably my most obfuscated expression.
        // Loop above looks for a "bubble". If there is no "bubble", that means, that
        // current_combination is the last combination, Expression in the if statement
        // below evaluates to true and the function exits returning result.
        // If the "bubble" is found however, the ststement below has a sideeffect of 
        // incrementing the first iterator to the left of the "bubble".
        if(++current_combination[i] == current_combination[i + 1u])
            return result;
        // Rest of the code sets posiotons of the rest of the iterstors
        // (if there are any), that are to the right of the incremented one,
        // to form next combination

        while(++i < combination_size)
        {
            current_combination[i] = current_combination[i - 1u];
            ++current_combination[i];
        }
        // Below is the ugly side of using the sentinel. Well it had to haave some 
        // disadvantage. Try without it.
        result.push_back(std::vector<Fci>(current_combination.begin(),
                                          current_combination.end() - 1));
    }
}

Plugin org.apache.maven.plugins:maven-clean-plugin:2.5 or one of its dependencies could not be resolved

For all the Windows users, try moving your codebase to a shorter windows path for eg: C:/myProj

Deeply nested Maven jar files can create a longer file path in windows. Since windows OS, by default, limits the file path length to 260 characters, it throws exception when trying to read a file located at a path that becomes more than 260 characters.

You can change this default to increase this limit to more than 260 chracters. Search around the web and you will find many posts on how to do that.

You can face similar problem while using npm packages also.

How do I add indices to MySQL tables?

It's worth noting that multiple field indexes can drastically improve your query performance. So in the above example we assume ProductID is the only field to lookup but were the query to say ProductID = 1 AND Category = 7 then a multiple column index helps. This is achieved with the following:

ALTER TABLE `table` ADD INDEX `index_name` (`col1`,`col2`)

Additionally the index should match the order of the query fields. In my extended example the index should be (ProductID,Category) not the other way around.

How can I pretty-print JSON using node.js?

Another workaround would be to make use of prettier to format the JSON. The example below is using 'json' parser but it could also use 'json5', see list of valid parsers.

const prettier = require("prettier");
console.log(prettier.format(JSON.stringify(object),{ semi: false, parser: "json" }));

Function or sub to add new row and data to table

I needed this same solution, but if you use the native ListObject.Add() method then you avoid the risk of clashing with any data immediately below the table. The below routine checks the last row of the table, and adds the data in there if it's blank; otherwise it adds a new row to the end of the table:

Sub AddDataRow(tableName As String, values() As Variant)
    Dim sheet As Worksheet
    Dim table As ListObject
    Dim col As Integer
    Dim lastRow As Range

    Set sheet = ActiveWorkbook.Worksheets("Sheet1")
    Set table = sheet.ListObjects.Item(tableName)

    'First check if the last row is empty; if not, add a row
    If table.ListRows.Count > 0 Then
        Set lastRow = table.ListRows(table.ListRows.Count).Range
        For col = 1 To lastRow.Columns.Count
            If Trim(CStr(lastRow.Cells(1, col).Value)) <> "" Then
                table.ListRows.Add
                Exit For
            End If
        Next col
    Else
        table.ListRows.Add
    End If

    'Iterate through the last row and populate it with the entries from values()
    Set lastRow = table.ListRows(table.ListRows.Count).Range
    For col = 1 To lastRow.Columns.Count
        If col <= UBound(values) + 1 Then lastRow.Cells(1, col) = values(col - 1)
    Next col
End Sub

To call the function, pass the name of the table and an array of values, one value per column. You can get / set the name of the table from the Design tab of the ribbon, in Excel 2013 at least: enter image description here

Example code for a table with three columns:

Dim x(2)
x(0) = 1
x(1) = "apple"
x(2) = 2
AddDataRow "Table1", x

CSS Div Background Image Fixed Height 100% Width

But the thing is that the .chapter class is not dynamic you're declaring a height:1200px

so it's better to use background:cover and set with media queries specific height's for popular resolutions.

How to set "value" to input web element using selenium?

As Shubham Jain stated, this is working to me: driver.findElement(By.id("invoice_supplier_id")).sendKeys("value"??, "new value");

How to enable C# 6.0 feature in Visual Studio 2013?

A lot of the answers here were written prior to Roslyn (the open-source .NET C# and VB compilers) moving to .NET 4.6. So they won't help you if your project targets, say, 4.5.2 as mine did (inherited and can't be changed).

But you can grab a previous version of Roslyn from https://www.nuget.org/packages/Microsoft.Net.Compilers and install that instead of the latest version. I used 1.3.2. (I tried 2.0.1 - which appears to be the last version that runs on .NET 4.5 - but I couldn't get it to compile*.) Run this from the Package Manager console in VS 2013:

PM> Install-Package Microsoft.Net.Compilers -Version 1.3.2

Then restart Visual Studio. I had a couple of problems initially; you need to set the C# version back to default (C#6.0 doesn't appear in the version list but seems to have been made the default), then clean, save, restart VS and recompile.

Interestingly, I didn't have any IntelliSense errors due to the C#6.0 features used in the code (which were the reason for wanting C#6.0 in the first place).

* version 2.0.1 threw error The "Microsoft.CodeAnalysis.BuildTasks.Csc task could not be loaded from the assembly Microsoft.Build.Tasks.CodeAnalysis.dll. Could not load file or assembly 'Microsoft.Build.Utilities.Core, Version=14.0.0.0, Culture=neutral, PublicKeyToken=b03f5f7f11d50a3a' or one of its dependencies. The system cannot find the file specified. Confirm that the declaration is correct, that the assembly and all its dependencies are available, and that the task contains a public class that implements Microsoft.Build.Framework.ITask.

UPDATE One thing I've noticed since posting this answer is that if you change any code during debug ("Edit and Continue"), you'll like find that your C#6.0 code will suddenly show as errors in what seems to revert to a pre-C#6.0 environment. This requires a restart of your debug session. VERY annoying especially for web applications.

Cordova : Requirements check failed for JDK 1.8 or greater

You don't have to uninstall any higher version of sdk. just install jdk1.8.0_161 or don't if it is already install.

Now just set the JAVA_HOME USER variable (not system variable) as shown in the below image.

enter image description here

This way you don't have to uninstall higher version and the problem get resolved.

Insert PHP code In WordPress Page and Post

When I was trying to accomplish something very similar, I ended up doing something along these lines:

wp-content/themes/resources/functions.php

add_action('init', 'my_php_function');
function my_php_function() {
    if (stripos($_SERVER['REQUEST_URI'], 'page-with-custom-php') !== false) {
        // add desired php code here
    }
}

afxwin.h file is missing in VC++ Express Edition

I encountered the same problem. The easiest thing is to install the free Visual Studio Community 2015 as answered in this question Is MFC only available with Visual Studio, and not Visual C++ Express?

Using <style> tags in the <body> with other HTML

When I see that the big-site Content Management Systems routinely put some <style> elements (some, not all) close to the content that relies on those classes, I conclude that the horse is out of the barn.

Go look at page sources from cnn.com, nytimes.com, huffingtonpost.com, your nearest big-city newspaper, etc. All of them do this.

If there's a good reason to put an extra <style> section somewhere in the body -- for instance if you're include()ing diverse and independent page elements in real time and each has an embedded <style> of its own, and the organization will be cleaner, more modular, more understandable, and more maintainable -- I say just bite the bullet. Sure it would be better if we could have "local" style with restricted scope, like local variables, but you go to work with the HTML you have, not the HTML you might want or wish to have at a later time.

Of course there are potential drawbacks and good (if not always compelling) reasons to follow the orthodoxy, as others have elaborated. But to me it looks more and more like thoughtful use of <style> in <body> has already gone mainstream.

How to write LaTeX in IPython Notebook?

Since, I was not able to use all the latex commands in Code even after using the %%latex keyword or the $..$ limiter, I installed the nbextensions through which I could use the latex commands in Markdown. After following the instructions here: https://github.com/ipython-contrib/IPython-notebook-extensions/blob/master/README.md and then restarting the Jupyter and then localhost:8888/nbextensions and then activating "Latex Environment for Jupyter", I could run many Latex commands. Examples are here: https://rawgit.com/jfbercher/latex_envs/master/doc/latex_env_doc.html

\section{First section}
\textbf{Hello}
$
\begin{equation} 
c = \sqrt{a^2 + b^2}
\end{equation}
$
\begin{itemize}
\item First item
\item Second item
\end{itemize}
\textbf{World}

As you see, I am still unable to use usepackage. But maybe it will be improved in the future.

enter image description here

CORS with spring-boot and angularjs not working

To build on other answers above, in case you have a Spring boot REST service application (not Spring MVC) with Spring security, then enabling CORS via Spring security is enough (if you use Spring MVC then using a WebMvcConfigurer bean as mentioned by Yogen could be the way to go as Spring security will delegate to the CORS definition mentioned therein)

So you need to have a security config that does the following:

@Configuration
@EnableWebSecurity
public class SecurityConfig extends WebSecurityConfigurerAdapter {

@Override
protected void configure(HttpSecurity http) throws Exception {
    //other http security config
    http.cors().configurationSource(corsConfigurationSource());
}

//This can be customized as required
CorsConfigurationSource corsConfigurationSource() {
    CorsConfiguration configuration = new CorsConfiguration();
    List<String> allowOrigins = Arrays.asList("*");
    configuration.setAllowedOrigins(allowOrigins);
    configuration.setAllowedMethods(singletonList("*"));
    configuration.setAllowedHeaders(singletonList("*"));
    //in case authentication is enabled this flag MUST be set, otherwise CORS requests will fail
    configuration.setAllowCredentials(true);
    UrlBasedCorsConfigurationSource source = new UrlBasedCorsConfigurationSource();
    source.registerCorsConfiguration("/**", configuration);
    return source;
}

}

This link has more information on the same: https://docs.spring.io/spring-security/site/docs/current/reference/htmlsingle/#cors

Note:

  1. Enabling CORS for all origins (*) for a prod deployed application may not always be a good idea.
  2. CSRF can be enabled via the Spring HttpSecurity customization without any issues
  3. In case you have authentication enabled in the app with Spring (via a UserDetailsService for example) then the configuration.setAllowCredentials(true); must be added

Tested for Spring boot 2.0.0.RELEASE (i.e., Spring 5.0.4.RELEASE and Spring security 5.0.3.RELEASE)

How to detect scroll position of page using jQuery

Now that works for me...

$(document).ready(function(){

    $(window).resize(function(e){
        console.log(e);                   
    });

    $(window).scroll(function (event) {
        var sc = $(window).scrollTop();
        console.log(sc);
    });

})

it works well... and then you can use JQuery/TweenMax to track elements and control them.

operator << must take exactly one argument

If you define operator<< as a member function it will have a different decomposed syntax than if you used a non-member operator<<. A non-member operator<< is a binary operator, where a member operator<< is a unary operator.

// Declarations
struct MyObj;
std::ostream& operator<<(std::ostream& os, const MyObj& myObj);

struct MyObj
{
    // This is a member unary-operator, hence one argument
    MyObj& operator<<(std::ostream& os) { os << *this; return *this; }

    int value = 8;
};

// This is a non-member binary-operator, 2 arguments
std::ostream& operator<<(std::ostream& os, const MyObj& myObj)
{
    return os << myObj.value;
}

So.... how do you really call them? Operators are odd in some ways, I'll challenge you to write the operator<<(...) syntax in your head to make things make sense.

MyObj mo;

// Calling the unary operator
mo << std::cout;

// which decomposes to...
mo.operator<<(std::cout);

Or you could attempt to call the non-member binary operator:

MyObj mo;

// Calling the binary operator
std::cout << mo;

// which decomposes to...
operator<<(std::cout, mo);

You have no obligation to make these operators behave intuitively when you make them into member functions, you could define operator<<(int) to left shift some member variable if you wanted to, understand that people may be a bit caught off guard, no matter how many comments you may write.

Almost lastly, there may be times where both decompositions for an operator call are valid, you may get into trouble here and we'll defer that conversation.

Lastly, note how odd it might be to write a unary member operator that is supposed to look like a binary operator (as you can make member operators virtual..... also attempting to not devolve and run down this path....)

struct MyObj
{
    // Note that we now return the ostream
    std::ostream& operator<<(std::ostream& os) { os << *this; return os; }

    int value = 8;
};

This syntax will irritate many coders now....

MyObj mo;

mo << std::cout << "Words words words";

// this decomposes to...
mo.operator<<(std::cout) << "Words words words";

// ... or even further ...
operator<<(mo.operator<<(std::cout), "Words words words");

Note how the cout is the second argument in the chain here.... odd right?

Remove leading and trailing spaces?

Expand your one liner into multiple lines. Then it becomes easy:

f.write(re.split("Tech ID:|Name:|Account #:",line)[-1])

parts = re.split("Tech ID:|Name:|Account #:",line)
wanted_part = parts[-1]
wanted_part_stripped = wanted_part.strip()
f.write(wanted_part_stripped)

Stretch image to fit full container width bootstrap

In bootstrap 4.1, the w-100 class is required along with img-fluid for images smaller than the page to be stretched:

<div class="container">
  <div class="row">
    <img class='img-fluid w-100' src="#" alt="" />
  </div>
</div>

see closed issue: https://github.com/twbs/bootstrap/issues/20830

(As of 2018-04-20, the documentation is wrong: https://getbootstrap.com/docs/4.1/content/images/ says that img-fluid applies max-width: 100%; height: auto;" but img-fluid does not resolve the issue, and neither does manually adding those style attributes with or without bootstrap classes on the img tag.)

Why doesn't git recognize that my file has been changed, therefore git add not working

I had this issue. Mine wasn't working because I was putting my files in the .git folder inside my project.

Read Session Id using Javascript

For PHP's PHPSESSID variable, this function works:

function getPHPSessId() {
    var phpSessionId = document.cookie.match(/PHPSESSID=[A-Za-z0-9]+\;/i);

    if(phpSessionId == null) 
        return '';

    if(typeof(phpSessionId) == 'undefined')
        return '';

    if(phpSessionId.length <= 0)
        return '';

    phpSessionId = phpSessionId[0];

    var end = phpSessionId.lastIndexOf(';');
    if(end == -1) end = phpSessionId.length;

    return phpSessionId.substring(10, end);
}

Fastest way to check if a value exists in a list

You could put your items into a set. Set lookups are very efficient.

Try:

s = set(a)
if 7 in s:
  # do stuff

edit In a comment you say that you'd like to get the index of the element. Unfortunately, sets have no notion of element position. An alternative is to pre-sort your list and then use binary search every time you need to find an element.

How can the Euclidean distance be calculated with NumPy?

Here's some concise code for Euclidean distance in Python given two points represented as lists in Python.

def distance(v1,v2): 
    return sum([(x-y)**2 for (x,y) in zip(v1,v2)])**(0.5)

How to center images on a web page for all screen sizes

In your specific case, you can set the containing a element to be:

a {
    display: block;
    text-align: center;
}

JS Bin demo.

How do I get the XML SOAP request of an WCF Web service request?

There is an another way to see XML SOAP - custom MessageEncoder. The main difference from IClientMessageInspector is that it works on lower level, so it captures original byte content including any malformed xml.

In order to implement tracing using this approach you need to wrap a standard textMessageEncoding with custom message encoder as new binding element and apply that custom binding to endpoint in your config.

Also you can see as example how I did it in my project - wrapping textMessageEncoding, logging encoder, custom binding element and config.

Disable vertical sync for glxgears

If you're using the NVIDIA closed-source drivers you can vary the vertical sync mode on the fly using the __GL_SYNC_TO_VBLANK environment variable:

~$ __GL_SYNC_TO_VBLANK=1 glxgears
Running synchronized to the vertical refresh.  The framerate should be
approximately the same as the monitor refresh rate.
299 frames in 5.0 seconds = 59.631 FPS

~$ __GL_SYNC_TO_VBLANK=0 glxgears
123259 frames in 5.0 seconds = 24651.678 FPS

This works for me on Ubuntu 14.04 using the 346.46 NVIDIA drivers.

How can I cast int to enum?

using System;
using System.Collections.Generic;
using System.Linq;
using System.Text.RegularExpressions;

namespace SamplePrograme
{
    public class Program
    {
        public enum Suit : int
        {
            Spades = 0,
            Hearts = 1,
            Clubs = 2,
            Diamonds = 3
        }

        public static void Main(string[] args)
        {
            //from string
            Console.WriteLine((Suit) Enum.Parse(typeof(Suit), "Clubs"));

            //from int
            Console.WriteLine((Suit)1);

            //From number you can also
            Console.WriteLine((Suit)Enum.ToObject(typeof(Suit) ,1));
        }
    }
}

What can <f:metadata>, <f:viewParam> and <f:viewAction> be used for?

Process GET parameters

The <f:viewParam> manages the setting, conversion and validation of GET parameters. It's like the <h:inputText>, but then for GET parameters.

The following example

<f:metadata>
    <f:viewParam name="id" value="#{bean.id}" />
</f:metadata>

does basically the following:

  • Get the request parameter value by name id.
  • Convert and validate it if necessary (you can use required, validator and converter attributes and nest a <f:converter> and <f:validator> in it like as with <h:inputText>)
  • If conversion and validation succeeds, then set it as a bean property represented by #{bean.id} value, or if the value attribute is absent, then set it as request attribtue on name id so that it's available by #{id} in the view.

So when you open the page as foo.xhtml?id=10 then the parameter value 10 get set in the bean this way, right before the view is rendered.

As to validation, the following example sets the param to required="true" and allows only values between 10 and 20. Any validation failure will result in a message being displayed.

<f:metadata>
    <f:viewParam id="id" name="id" value="#{bean.id}" required="true">
        <f:validateLongRange minimum="10" maximum="20" />
    </f:viewParam>
</f:metadata>
<h:message for="id" />

Performing business action on GET parameters

You can use the <f:viewAction> for this.

<f:metadata>
    <f:viewParam id="id" name="id" value="#{bean.id}" required="true">
        <f:validateLongRange minimum="10" maximum="20" />
    </f:viewParam>
    <f:viewAction action="#{bean.onload}" />
</f:metadata>
<h:message for="id" />

with

public void onload() {
    // ...
}

The <f:viewAction> is however new since JSF 2.2 (the <f:viewParam> already exists since JSF 2.0). If you can't upgrade, then your best bet is using <f:event> instead.

<f:event type="preRenderView" listener="#{bean.onload}" />

This is however invoked on every request. You need to explicitly check if the request isn't a postback:

public void onload() {
    if (!FacesContext.getCurrentInstance().isPostback()) {
        // ...
    }
}

When you would like to skip "Conversion/Validation failed" cases as well, then do as follows:

public void onload() {
    FacesContext facesContext = FacesContext.getCurrentInstance();
    if (!facesContext.isPostback() && !facesContext.isValidationFailed()) {
        // ...
    }
}

Using <f:event> this way is in essence a workaround/hack, that's exactly why the <f:viewAction> was introduced in JSF 2.2.


Pass view parameters to next view

You can "pass-through" the view parameters in navigation links by setting includeViewParams attribute to true or by adding includeViewParams=true request parameter.

<h:link outcome="next" includeViewParams="true">
<!-- Or -->
<h:link outcome="next?includeViewParams=true">

which generates with the above <f:metadata> example basically the following link

<a href="next.xhtml?id=10">

with the original parameter value.

This approach only requires that next.xhtml has also a <f:viewParam> on the very same parameter, otherwise it won't be passed through.


Use GET forms in JSF

The <f:viewParam> can also be used in combination with "plain HTML" GET forms.

<f:metadata>
    <f:viewParam id="query" name="query" value="#{bean.query}" />
    <f:viewAction action="#{bean.search}" />
</f:metadata>
...
<form>
    <label for="query">Query</label>
    <input type="text" name="query" value="#{empty bean.query ? param.query : bean.query}" />
    <input type="submit" value="Search" />
    <h:message for="query" />
</form>
...
<h:dataTable value="#{bean.results}" var="result" rendered="#{not empty bean.results}">
     ...
</h:dataTable>

With basically this @RequestScoped bean:

private String query;
private List<Result> results;

public void search() {
    results = service.search(query);
}

Note that the <h:message> is for the <f:viewParam>, not the plain HTML <input type="text">! Also note that the input value displays #{param.query} when #{bean.query} is empty, because the submitted value would otherwise not show up at all when there's a validation or conversion error. Please note that this construct is invalid for JSF input components (it is doing that "under the covers" already).


See also:

Is there a stopwatch in Java?

The code doesn't work because elapsed variable in getElapsedTimeSecs() is not a float or double.

Set margins in a LinearLayout programmatically

Due to variation in device screen pixel densities its good to always use DIP unit to set margin programmatically. Like below_

//get resources
Resources r = getResources();
float pxLeftMargin = TypedValue.applyDimension(TypedValue.COMPLEX_UNIT_DIP, 10, r.getDisplayMetrics());
float pxTopMargin = TypedValue.applyDimension(TypedValue.COMPLEX_UNIT_DIP, 10, r.getDisplayMetrics());
float pxRightMargin = TypedValue.applyDimension(TypedValue.COMPLEX_UNIT_DIP, 10, r.getDisplayMetrics());
float pxBottomMargin = TypedValue.applyDimension(TypedValue.COMPLEX_UNIT_DIP, 20, r.getDisplayMetrics());

//get layout params...
LayoutParams params=new LayoutParams(LayoutParams.MATCH_PARENT, LayoutParams.MATCH_PARENT);
params.setMargins(Math.round(pxLeftMargin), Math.round(pxTopMargin), Math.round(pxRightMargin), Math.round(pxBottomMargin));

//set margin...
yourLayoutTOsetMargin.setLayoutParams(params); 

Hope this will help.

Angular 4 - get input value

You can also use template reference variables

<form (submit)="onSubmit(player.value)">
   <input #player placeholder="player name">
</form>
onSubmit(playerName: string) {
  console.log(playerName)
}

Convert file: Uri to File in Android

For this case, especially on Android, the way going for bytes is usually faster.

With this, I solved it by setting up a class FileHelper which is given the responsibility to deal with reading/writing bytes from/to file through stream and a class UriHelper which is given the responsibility to figure out path of Uri and permission.

As far as it's knew generally, string.getBytes((charset == null) ? DEFAULT_CHARSET:charset) can help us to transfer string you want to bytes you need.

How to let UriHelper and FileHelper you to copy a picture noted by Uri into a file, you can run:

FileHelper.getInstance().copy(UriHelper.getInstance().toFile(uri_of_a_picture)
                        , FileHelper.getInstance().createExternalFile(null, UriHelper.getInstance().generateFileNameBasedOnTimeStamp()
                                + UriHelper.getInstance().getFileName(uri_of_a_picture, context), context)
                );

about my UriHelper:

public class UriHelper {
private static UriHelper INSTANCE = new UriHelper();

public static UriHelper getInstance() {
    return INSTANCE;
}

@SuppressLint("SimpleDateFormat")
public String generateFileNameBasedOnTimeStamp() {
    return new SimpleDateFormat("yyyyMMdd_hhmmss").format(new Date()) + ".jpeg";
}

/**
 * if uri.getScheme.equals("content"), open it with a content resolver.
 * if the uri.Scheme.equals("file"), open it using normal file methods.
 */
//

public File toFile(Uri uri) {
    if (uri == null) return null;
    Logger.d(">>> uri path:" + uri.getPath());
    Logger.d(">>> uri string:" + uri.toString());
    return new File(uri.getPath());
}

public DocumentFile toDocumentFile(Uri uri) {
    if (uri == null) return null;
    Logger.d(">>> uri path:" + uri.getPath());
    Logger.d(">>> uri string:" + uri.toString());
    return DocumentFile.fromFile(new File(uri.getPath()));
}

public Uri toUri(File file) {
    if (file == null) return null;
    Logger.d(">>> file path:" + file.getAbsolutePath());
    return Uri.fromFile(file); //returns an immutable URI reference representing the file
}

public String getPath(Uri uri, Context context) {
    if (uri == null) return null;
    if (uri.getScheme() == null) return null;
    Logger.d(">>> uri path:" + uri.getPath());
    Logger.d(">>> uri string:" + uri.toString());
    String path;
    if (uri.getScheme().equals("content")) {
        //Cursor cursor = context.getContentResolver().query(uri, new String[] {MediaStore.Images.ImageColumns.DATA}, null, null, null);
        Cursor cursor = context.getContentResolver().query(uri, null, null, null, null);
        if (cursor == null) {
            Logger.e("!!! cursor is null");
            return null;
        }
        if (cursor.getCount() >= 0) {
            Logger.d("... the numbers of rows:" + cursor.getCount()
                        + "and the numbers of columns:" + cursor.getColumnCount());
            if (cursor.isBeforeFirst()) {
                while (cursor.moveToNext()) {
                    StringBuilder stringBuilder = new StringBuilder();
                    for (int i = 0; i<cursor.getColumnCount(); i++) {
                        stringBuilder.append("... iterating cursor.getString(" + i +"(" + cursor.getColumnName(i) + ")):" + cursor.getString(i));
                        stringBuilder.append("\n");
                    }
                    Logger.d(stringBuilder.toString());
                }
            } else {
                cursor.moveToFirst();
                do {
                    StringBuilder stringBuilder = new StringBuilder();
                    for (int i = 0; i<cursor.getColumnCount(); i++) {
                        stringBuilder.append("... iterating cursor.getString(" + i +"(" + cursor.getColumnName(i) + ")):" + cursor.getString(i));
                        stringBuilder.append("\n");
                    }
                    Logger.d(stringBuilder.toString());
                } while (cursor.moveToNext());
            }
            path = uri.getPath();
            cursor.close();
            Logger.d("... content scheme:" + uri.getScheme() + "  and return:" + path);
            return path;
        } else {
            path = uri.getPath();
            Logger.d("... content scheme:" + uri.getScheme()
                    + " but the numbers of rows in the cursor is < 0:" + cursor.getCount()
                    + "  and return:" + path);
            return path;
        }
    } else {
        path = uri.getPath();
        Logger.d("... not content scheme:" + uri.getScheme() + "  and return:" + path);
        return path;
    }
}

public String getFileName(Uri uri, Context context) {
    if (uri == null) return null;
    if (uri.getScheme() == null) return null;
    Logger.d(">>> uri path:" + uri.getPath());
    Logger.d(">>> uri string:" + uri.toString());
    String path;
    if (uri.getScheme().equals("content")) {
        //Cursor cursor = context.getContentResolver().query(uri, new String[] {MediaStore.Images.ImageColumns.DATA}, null, null, null);
        Cursor cursor = context.getContentResolver().query(uri, null, null, null, null);
        if (cursor == null) {
            Logger.e("!!! cursor is null");
            return null;
        }
        if (cursor.getCount() >= 0) {
            Logger.d("... the numbers of rows:" + cursor.getCount()
                    + "and the numbers of columns:" + cursor.getColumnCount());
            if (cursor.isBeforeFirst()) {
                while (cursor.moveToNext()) {
                    StringBuilder stringBuilder = new StringBuilder();
                    for (int i = 0; i<cursor.getColumnCount(); i++) {
                        stringBuilder.append("... iterating cursor.getString(" + i +"(" + cursor.getColumnName(i) + ")):" + cursor.getString(i));
                        stringBuilder.append("\n");
                    }
                    Logger.d(stringBuilder.toString());
                }
            } else {
                cursor.moveToFirst();
                do {
                    StringBuilder stringBuilder = new StringBuilder();
                    for (int i = 0; i<cursor.getColumnCount(); i++) {
                        stringBuilder.append("... iterating cursor.getString(" + i +"(" + cursor.getColumnName(i) + ")):" + cursor.getString(i));
                        stringBuilder.append("\n");
                    }
                    Logger.d(stringBuilder.toString());
                } while (cursor.moveToNext());
            }
            cursor.moveToFirst();
            path = cursor.getString(cursor.getColumnIndex(MediaStore.Images.ImageColumns.DISPLAY_NAME));
            cursor.close();
            Logger.d("... content scheme:" + uri.getScheme() + "  and return:" + path);
            return path;
        } else {
            path = uri.getLastPathSegment();
            Logger.d("... content scheme:" + uri.getScheme()
                    + " but the numbers of rows in the cursor is < 0:" + cursor.getCount()
                    + "  and return:" + path);
            return path;
        }
    } else {
        path = uri.getLastPathSegment();
        Logger.d("... not content scheme:" + uri.getScheme() + "  and return:" + path);
        return path;
    }
}

}

about my FileHelper:

public class FileHelper {
private static final String DEFAULT_DIR_NAME = "AmoFromTaiwan";
private static final int DEFAULT_BUFFER_SIZE = 1024;
private static final Charset DEFAULT_CHARSET = Charset.forName("UTF-8");
private static final int EOF = -1;
private static FileHelper INSTANCE = new FileHelper();

public static FileHelper getInstance() {
    return INSTANCE;
}

private boolean isExternalStorageWritable(Context context) {
    /*
    String state = Environment.getExternalStorageState();
    return Environment.MEDIA_MOUNTED.equals(state);
    */
    if (Build.VERSION.SDK_INT >= Build.VERSION_CODES.M) {
        if (context.checkSelfPermission(android.Manifest.permission.WRITE_EXTERNAL_STORAGE) == PackageManager.PERMISSION_GRANTED) {
            return true;
        } else {
            Logger.e("!!! checkSelfPermission() not granted");
            return false;
        }
    } else { //permission is automatically granted on sdk<23 upon installation
        return true;
    }
}

private boolean isExternalStorageReadable(Context context) {
    if (Build.VERSION.SDK_INT >= Build.VERSION_CODES.M) {
        if (context.checkSelfPermission(android.Manifest.permission.READ_EXTERNAL_STORAGE) == PackageManager.PERMISSION_GRANTED) {
            return true;
        } else {
            Logger.e("!!! checkSelfPermission() not granted");
            return false;
        }
    } else { //permission is automatically granted on sdk<23 upon installation
        return true;
    }
}

@SuppressLint("SimpleDateFormat")
private String generateFileNameBasedOnTimeStamp() {
    return new SimpleDateFormat("yyyyMMdd_hhmmss").format(new Date()) + ".jpeg";
}

public File createExternalFile(String dir_name, String file_name, Context context) {
    String dir_path;
    String file_path;
    File dir ;
    File file;
    if (!isExternalStorageWritable(context)) {
        Logger.e("!!! external storage not writable");
        return null;
    }
    if (dir_name == null) {
        dir_path = Environment.getExternalStoragePublicDirectory(Environment.DIRECTORY_PICTURES).getAbsolutePath() + File.separator + DEFAULT_DIR_NAME;
    } else {
        dir_path = Environment.getExternalStoragePublicDirectory(Environment.DIRECTORY_PICTURES).getAbsolutePath() + File.separator + dir_name;
    }
    Logger.d("... going to access an external dir:" + dir_path);
    dir = new File(dir_path);
    if (!dir.exists()) {
        Logger.d("... going to mkdirs:" + dir_path);
        if (!dir.mkdirs()) {
            Logger.e("!!! failed to mkdirs");
            return null;
        }
    }
    if (file_name == null) {
        file_path = dir_path + File.separator + generateFileNameBasedOnTimeStamp();
    } else {
        file_path = dir_path + File.separator + file_name;
    }
    Logger.d("... going to return an external dir:" + file_path);
    file = new File(file_path);
    if (file.exists()) {
        Logger.d("... before creating to delete an external dir:" + file.getAbsolutePath());
        if (!file.delete()) {
            Logger.e("!!! failed to delete file");
            return null;
        }
    }
    return file;
}

public File createInternalFile(String dir_name, String file_name, Context context) {
    String dir_path;
    String file_path;
    File dir ;
    File file;
    if (dir_name == null) {
        dir = new ContextWrapper(context).getDir(DEFAULT_DIR_NAME, Context.MODE_PRIVATE);
    } else {
        dir = new ContextWrapper(context).getDir(dir_name, Context.MODE_PRIVATE);
    }
    dir_path = dir.getAbsolutePath();
    Logger.d("... going to access an internal dir:" + dir_path);
    if (!dir.exists()) {
        Logger.d("... going to mkdirs:" + dir_path);
        if (!dir.mkdirs()) {
            Logger.e("!!! mkdirs failed");
            return null;
        }
    }
    if (file_name == null) {
        file = new File(dir, generateFileNameBasedOnTimeStamp());
    } else {
        file = new File(dir, file_name);
    }
    file_path = file.getAbsolutePath();
    Logger.d("... going to return an internal dir:" + file_path);
    if (file.exists()) {
        Logger.d("... before creating to delete an external dir:" + file.getAbsolutePath());
        if (!file.delete()) {
            Logger.e("!!! failed to delete file");
            return null;
        }
    }
    return file;
}

public File getExternalFile(String dir_name, String file_name, Context context) {
    String dir_path;
    String file_path;
    File file;
    if (!isExternalStorageWritable(context)) {
        Logger.e("!!! external storage not writable");
        return null;
    }
    if (dir_name == null) {
        dir_path = Environment.getExternalStoragePublicDirectory(Environment.DIRECTORY_PICTURES).getAbsolutePath() + File.separator + DEFAULT_DIR_NAME;
    } else {
        dir_path = Environment.getExternalStoragePublicDirectory(Environment.DIRECTORY_PICTURES).getAbsolutePath() + File.separator + dir_name;
    }
    if (file_name == null) {
        file_path = dir_path;
    } else {
        file_path = dir_path + File.separator + file_name;
    }
    Logger.d("... going to return an external file:" + file_path);
    file = new File(file_path);
    if (file.exists()) {
        Logger.d("... file exists:" + file.getAbsolutePath());
    } else {
        Logger.e("!!! file does't exist:" + file.getAbsolutePath());
    }
    return file;
}

public File getInternalFile(String dir_name, String file_name, Context context) {
    String file_path;
    File dir ;
    File file;
    if (dir_name == null) {
        dir = new ContextWrapper(context).getDir(DEFAULT_DIR_NAME, Context.MODE_PRIVATE);
    } else {
        dir = new ContextWrapper(context).getDir(dir_name, Context.MODE_PRIVATE);
    }
    if (file_name == null) {
        file = new File(dir.getAbsolutePath());
    } else {
        file = new File(dir, file_name);
    }
    file_path = file.getAbsolutePath();
    Logger.d("... going to return an internal dir:" + file_path);
    if (file.exists()) {
        Logger.d("... file exists:" + file.getAbsolutePath());
    } else {
        Logger.e("!!! file does't exist:" + file.getAbsolutePath());
    }
    return file;
}

private byte[] readBytesFromFile(File file) {
    Logger.d(">>> path:" + file.getAbsolutePath());
    FileInputStream fis;
    long file_length;
    byte[] buffer;
    int offset = 0;
    int next = 0;
    if (!file.exists()) {
        Logger.e("!!! file doesn't exists");
        return null;
    }
    if (file.length() > Integer.MAX_VALUE) {
        Logger.e("!!! file length is out of max of int");
        return null;
    } else {
        file_length = file.length();
    }
    try {
        fis = new FileInputStream(file);
        //buffer = new byte[(int) file_length];
        buffer = new byte[(int) file.length()];
        long time_start = System.currentTimeMillis();
        while (true) {
            Logger.d("... now next:" + next + " and offset:" + offset);
            if (System.currentTimeMillis() - time_start > 1000) {
                Logger.e("!!! left due to time out");
                break;
            }
            next = fis.read(buffer, offset, (buffer.length-offset));
            if (next < 0 || offset >= buffer.length) {
                Logger.d("... completed to read");
                break;
            }
            offset += next;
        }
        //if (offset < buffer.length) {
        if (offset < (int) file_length) {
            Logger.e("!!! not complete to read");
            return null;
        }
        fis.close();
        return buffer;
    } catch (IOException e) {
        e.printStackTrace();
        Logger.e("!!! IOException");
        return null;
    }
}

public byte[] readBytesFromFile(File file, boolean is_fis_fos_only) {
    if (file == null) return null;
    if (is_fis_fos_only) {
        return readBytesFromFile(file);
    }
    Logger.d(">>> path:" + file.getAbsolutePath());
    FileInputStream fis;
    BufferedInputStream bis;
    ByteArrayOutputStream bos;
    byte[] buf = new byte[(int) file.length()];
    int num_read;
    if (!file.exists()) {
        Logger.e("!!! file doesn't exists");
        return null;
    }
    try {
        fis = new FileInputStream(file);
        bis = new BufferedInputStream(fis);
        bos = new ByteArrayOutputStream();
        long time_start = System.currentTimeMillis();
        while (true) {
            if (System.currentTimeMillis() - time_start > 1000) {
                Logger.e("!!! left due to time out");
                break;
            }
            num_read = bis.read(buf, 0, buf.length); //1024 bytes per call
            if (num_read < 0) break;
            bos.write(buf, 0, num_read);
        }
        buf = bos.toByteArray();
        fis.close();
        bis.close();
        bos.close();
        return buf;
    } catch (FileNotFoundException e) {
        e.printStackTrace();
        Logger.e("!!! FileNotFoundException");
        return null;
    } catch (IOException e) {
        e.printStackTrace();
        Logger.e("!!! IOException");
        return null;
    }
}

/**
 * streams (InputStream and OutputStream) transfer binary data
 * if to write a string to a stream, must first convert it to bytes, or in other words encode it
 */
public boolean writeStringToFile(File file, String string, Charset charset) {
    if (file == null) return false;
    if (string == null) return false;
    return writeBytesToFile(file, string.getBytes((charset == null) ? DEFAULT_CHARSET:charset));
}

public boolean writeBytesToFile(File file, byte[] data) {
    if (file == null) return false;
    if (data == null) return false;
    FileOutputStream fos;
    BufferedOutputStream bos;
    try {
        fos = new FileOutputStream(file);
        bos = new BufferedOutputStream(fos);
        bos.write(data, 0, data.length);
        bos.flush();
        bos.close();
        fos.close();
    } catch (IOException e) {
        e.printStackTrace();
        Logger.e("!!! IOException");
        return false;
    }
    return true;
}

/**
 * io blocks until some input/output is available.
 */
public boolean copy(File source, File destination) {
    if (source == null || destination == null) return false;
    Logger.d(">>> source:" + source.getAbsolutePath() + ", destination:" + destination.getAbsolutePath());
    try {
        FileInputStream fis = new FileInputStream(source);
        FileOutputStream fos = new FileOutputStream(destination);
        byte[] buffer = new byte[(int) source.length()];
        int len;
        while (EOF != (len = fis.read(buffer))) {
            fos.write(buffer, 0, len);
        }
        if (true) { //debug
            byte[] copies = readBytesFromFile(destination);
            if (copies != null) {
                int copy_len = copies.length;
                Logger.d("... stream read and write done for " + copy_len + " bytes");
            }
        }
        return destination.length() != 0;
    } catch (IOException e) {
        e.printStackTrace();
        return false;
    }
}

public void list(final String path, final String end, final List<File> files) {
    Logger.d(">>> path:" + path + ", end:" + end);
    File file = new File(path);
    if (file.isDirectory()) {
        for (File child : file.listFiles()){
            list(child.getAbsolutePath(), end, files);
        }
    } else if (file.isFile()) {
        if (end.equals("")) {
            files.add(file);
        } else {
            if (file.getName().endsWith(end)) files.add(file);
        }
    }
}

public String[] splitFileName(File file, String split) {
    String path;
    String ext;
    int lastIndexOfSplit = file.getAbsolutePath().lastIndexOf(split);
    if (lastIndexOfSplit < 0) {
        path = file.getAbsolutePath();
        ext = "";
    } else {
        path = file.getAbsolutePath().substring(0, lastIndexOfSplit);
        ext = file.getAbsolutePath().substring(lastIndexOfSplit);
    }
    return new String[] {path, ext};
}

public File rename(File old_file, String new_name) {
    if (old_file == null || new_name == null) return null;
    Logger.d(">>> old file path:" + old_file.getAbsolutePath() + ", new file name:" + new_name);
    File new_file = new File(old_file, new_name);
    if (!old_file.equals(new_file)) {
        if (new_file.exists()) { //if find out previous file/dir at new path name exists
            if (new_file.delete()) {
                Logger.d("... succeeded to delete previous file at new abstract path name:" + new_file.getAbsolutePath());
            } else {
                Logger.e("!!! failed to delete previous file at new abstract path name");
                return null;
            }
        }
        if (old_file.renameTo(new_file)) {
            Logger.d("... succeeded to rename old file to new abstract path name:" + new_file.getAbsolutePath());
        } else {
            Logger.e("!!! failed to rename old file to new abstract path name");
        }
    } else {
        Logger.d("... new and old file have the equal abstract path name:" + new_file.getAbsolutePath());
    }
    return new_file;
}

public boolean remove(final String path, final String end) {
    Logger.d(">>> path:" + path + ", end:" + end);
    File file = new File(path);
    boolean result = false;
    if (file.isDirectory()) {
        for (File child : file.listFiles()){
            result = remove(child.getAbsolutePath(), end);
        }
    } else if (file.isFile()) {
        if (end.equals("")) {
            result = file.delete();
        } else {
            if (file.getName().endsWith(end)) result = file.delete();
        }
    } else {
        Logger.e("!!! child is not file or directory");
    }
    return result;
}

@TargetApi(Build.VERSION_CODES.O)
public byte[] readNIOBytesFromFile(String path) throws IOException {
    Logger.d(">>> path:" + path);
    if (!Files.exists(Paths.get(path), LinkOption.NOFOLLOW_LINKS)) {
        Logger.e("!!! file doesn't exists");
        return null;
    } else {
        return Files.readAllBytes(Paths.get(path));
    }
}

@TargetApi(Build.VERSION_CODES.O)
public File writeNIOBytesToFile(String dir, String name, byte[] data) {
    Logger.d(">>> dir:" + dir + ", name:" + name);
    Path path_dir;
    Path path_file;
    try {
        if (!Files.exists(Paths.get(dir), LinkOption.NOFOLLOW_LINKS)) {
            Logger.d("... make a dir");
            path_dir = Files.createDirectories(Paths.get(dir));
            if (path_dir == null) {
                Logger.e("!!! failed to make a dir");
                return null;
            }
        }
        path_file = Files.write(Paths.get(name), data);
        return path_file.toFile();
    } catch (IOException e) {
        e.printStackTrace();
        Logger.e("!!! IOException");
        return null;
    }
}

@TargetApi(Build.VERSION_CODES.O)
public void listNIO(final String dir, final String end, final List<File> files) throws IOException {
    Logger.d(">>> dir:" + dir + ", end:" + end);
    Files.walkFileTree(Paths.get(dir), new FileVisitor<Path>() {
        @Override
        public FileVisitResult preVisitDirectory(Path dir, BasicFileAttributes attrs) {
            Logger.d("... file:" + dir.getFileName());
            return FileVisitResult.CONTINUE;
        }

        @Override
        public FileVisitResult visitFile(Path file, BasicFileAttributes attrs) {
            Logger.d("... file:" + file.getFileName());
            if (end.equals("")) {
                files.add(file.toFile());
            } else {
                if (file.endsWith(end)) files.add(file.toFile());
            }
            return FileVisitResult.CONTINUE;
        }

        @Override
        public FileVisitResult visitFileFailed(Path file, IOException exc) {
            Logger.d("... file:" + file.getFileName());
            if (end.equals("")) {
                files.add(file.toFile());
            } else {
                if (file.endsWith(end)) files.add(file.toFile());
            }
            return FileVisitResult.CONTINUE;
        }

        @Override
        public FileVisitResult postVisitDirectory(Path dir, IOException exc) {
            Logger.d("... file:" + dir.getFileName());
            return FileVisitResult.CONTINUE;
        }
    });
}

/**
 * recursion
 */
private int factorial (int x) {
    if (x > 1) return (x*(factorial(x-1)));
    else if (x == 1) return x;
    else return 0;
}

}

Loading context in Spring using web.xml

You can also load the context while defining the servlet itself (WebApplicationContext)

  <servlet>
    <servlet-name>admin</servlet-name>
    <servlet-class>org.springframework.web.servlet.DispatcherServlet</servlet-class>
    <init-param>
      <param-name>contextConfigLocation</param-name>
      <param-value>
                /WEB-INF/spring/*.xml
            </param-value>
    </init-param>
    <load-on-startup>1</load-on-startup>
  </servlet>
  <servlet-mapping>
    <servlet-name>admin</servlet-name>
    <url-pattern>/</url-pattern>
  </servlet-mapping>

rather than (ApplicationContext)

<context-param>
    <param-name>contextConfigLocation</param-name>
    <param-value>/WEB-INF/applicationContext*.xml</param-value>
</context-param>

<listener>
   <listener-class>
        org.springframework.web.context.ContextLoaderListener
   </listener-class>
</listener> 

or can do both together.

Drawback of just using WebApplicationContext is that it will load context only for this particular Spring entry point (DispatcherServlet) where as with above mentioned methods context will be loaded for multiple entry points (Eg. Webservice Servlet, REST servlet etc)

Context loaded by ContextLoaderListener will infact be a parent context to that loaded specifically for DisplacherServlet . So basically you can load all your business service, data access or repository beans in application context and separate out your controller, view resolver beans to WebApplicationContext.

htaccess <Directory> deny from all

You can use from root directory:

RewriteEngine On
RewriteRule ^(?:system)\b.* /403.html

Or:

RewriteRule ^(?:system)\b.* /403.php # with header('HTTP/1.0 403 Forbidden');

How to write oracle insert script with one field as CLOB?

Keep in mind that SQL strings can not be larger than 4000 bytes, while Pl/SQL can have strings as large as 32767 bytes. see below for an example of inserting a large string via an anonymous block which I believe will do everything you need it to do.

note I changed the varchar2(32000) to CLOB

set serveroutput ON 
CREATE TABLE testclob 
  ( 
     id NUMBER, 
     c  CLOB, 
     d  VARCHAR2(4000) 
  ); 

DECLARE 
    reallybigtextstring CLOB := '123'; 
    i                   INT; 
BEGIN 
    WHILE Length(reallybigtextstring) <= 60000 LOOP 
        reallybigtextstring := reallybigtextstring 
                               || '000000000000000000000000000000000'; 
    END LOOP; 

    INSERT INTO testclob 
                (id, 
                 c, 
                 d) 
    VALUES     (0, 
                reallybigtextstring, 
                'done'); 

    dbms_output.Put_line('I have finished inputting your clob: ' 
                         || Length(reallybigtextstring)); 
END; 

/ 
SELECT * 
FROM   testclob; 


 "I have finished inputting your clob: 60030"

Swift do-try-catch syntax

I was also disappointed by the lack of type a function can throw, but I get it now thanks to @rickster and I'll summarize it like this: let's say we could specify the type a function throws, we would have something like this:

enum MyError: ErrorType { case ErrorA, ErrorB }

func myFunctionThatThrows() throws MyError { ...throw .ErrorA...throw .ErrorB... }

do {
    try myFunctionThatThrows()
}
case .ErrorA { ... }
case .ErrorB { ... }

The problem is that even if we don't change anything in myFunctionThatThrows, if we just add an error case to MyError:

enum MyError: ErrorType { case ErrorA, ErrorB, ErrorC }

we are screwed because our do/try/catch is no longer exhaustive, as well as any other place where we called functions that throw MyError

How to synchronize or lock upon variables in Java?

For this functionality you are better off not using a lock at all. Try an AtomicReference.

public class Sample {
    private final AtomicReference<String> msg = new AtomicReference<String>();

    public void setMsg(String x) {
        msg.set(x);
    }

    public String getMsg() {
        return msg.getAndSet(null);
    }
}

No locks required and the code is simpler IMHO. In any case, it uses a standard construct which does what you want.

Install python 2.6 in CentOS

Late to the party, but the OP should have gone with Buildout or Virtualenv, and sidestepped the problem completely.

I am currently working on a Centos server, well, toiling away would be the proper term and I can assure everyone that the only way I am able to blink back the tears whilst using the software equivalents of fire hardened spears, is buildout.

How to update TypeScript to latest version with npm?

If you are using Windows with very old NodeJS, then uninstall previous NodeJs and NVM (Node Version Manager) in Control Panel (Win7) or Settings/Apps (Win10) if exists. Make sure that they are removed from the PATH.

Reinstall NodeJS: https://nodejs.org/en/download It will install NPM as well.

Install TypeScript globally:

npm install -g typescript

Verify installation:

tsc -v

How can I select from list of values in SQL Server

Simplest way to get the distinct values of a long list of comma delimited text would be to use a find an replace with UNION to get the distinct values.

SELECT 1
UNION SELECT 1
UNION SELECT 1
UNION SELECT 2
UNION SELECT 5
UNION SELECT 1
UNION SELECT 6

Applied to your long line of comma delimited text

  • Find and replace every comma with UNION SELECT
  • Add a SELECT in front of the statement

You now should have a working query

Is it .yaml or .yml?

The nature and even existence of file extensions is platform-dependent (some obscure platforms don't even have them, remember) -- in other systems they're only conventional (UNIX and its ilk), while in still others they have definite semantics and in some cases specific limits on length or character content (Windows, etc.).

Since the maintainers have asked that you use ".yaml", that's as close to an "official" ruling as you can get, but the habit of 8.3 is hard to get out of (and, appallingly, still occasionally relevant in 2013).

Failed to run sdkmanager --list with Java 9

Update 2019-10:

As stated in the issue tracker, Google has been working on a new SDK tools release that runs on current JVMs (9+)!

You can download and use the new Android SDK Command-line Tools (1.0.0) inside Android Studio or by manually downloading them from the Google servers:

For the latest versions check the URLs inside the repository.xml.

If you manually unpack the command line tools, take care of placing them in a subfolder inside your $ANDROID_HOME (e.g. $ANDROID_HOME/cmdline-tools/...).

Is it possible to convert char[] to char* in C?

You don't need to declare them as arrays if you want to use use them as pointers. You can simply reference pointers as if they were multi-dimensional arrays. Just create it as a pointer to a pointer and use malloc:

int i;
int M=30, N=25;
int ** buf;
buf = (int**) malloc(M * sizeof(int*));
for(i=0;i<M;i++)
    buf[i] = (int*) malloc(N * sizeof(int));

and then you can reference buf[3][5] or whatever.

Naming Classes - How to avoid calling everything a "<WhatEver>Manager"?

I believe the critical thing here is to be consistent within the sphere of your code's visibility, i.e. as long as everyone who needs to look at/work on your code understands your naming convention then that should be fine, even if you decide to call them 'CompanyThingamabob' and 'UserDoohickey'. The first stop, if you work for a company, is to see if there is a company convention for naming. If there isn't or you don't work for a company then create your own using terms that make sense to you, pass it around a few trusted colleagues/friends who at least code casually, and incorporate any feedback that makes sense.

Applying someone else's convention, even when it's widely accepted, if it doesn't leap off the page at you is a bit of a mistake in my book. First and foremost I need to understand my code without reference to other documentation but at the same time it needs to be generic enough that it's no incomprehensible to someone else in the same field in the same industry.

How can I put a ListView into a ScrollView without it collapsing?

A solution I use is, to add all Content of the ScrollView (what should be above and under the listView) as headerView and footerView in the ListView.

So it works like, also the convertview is resued how it should be.

Generate random numbers following a normal distribution in C/C++

Use std::tr1::normal_distribution.

The std::tr1 namespace is not a part of boost. It's the namespace that contains the library additions from the C++ Technical Report 1 and is available in up to date Microsoft compilers and gcc, independently of boost.

Multi-dimensional arraylist or list in C#?

You can create a list of lists

   public class MultiDimList: List<List<string>> {  }

or a Dictionary of key-accessible Lists

   public class MultiDimDictList: Dictionary<string, List<int>>  { }
   MultiDimDictList myDicList = new MultiDimDictList ();
   myDicList.Add("ages", new List<int>()); 
   myDicList.Add("Salaries", new List<int>()); 
   myDicList.Add("AccountIds", new List<int>()); 

Generic versions, to implement suggestion in comment from @user420667

  public class MultiDimList<T>: List<List<T>> {  }

and for the dictionary,

   public class MultiDimDictList<K, T>: Dictionary<K, List<T>>  { }

  // to use it, in client code
   var myDicList = new MultiDimDictList<string, int> ();
   myDicList.Add("ages", new List<T>()); 
   myDicList["ages"].Add(23);
   myDicList["ages"].Add(32);
   myDicList["ages"].Add(18);

   myDicList.Add("salaries", new List<T>());
   myDicList["salaries"].Add(80000);
   myDicList["salaries"].Add(100000);

   myDicList.Add("accountIds", new List<T>()); 
   myDicList["accountIds"].Add(321123);
   myDicList["accountIds"].Add(342653);

or, even better, ...

   public class MultiDimDictList<K, T>: Dictionary<K, List<T>>  
   {
       public void Add(K key, T addObject)
       {
           if(!ContainsKey(key)) Add(key, new List<T>());
           if (!base[key].Contains(addObject)) base[key].Add(addObject);
       }           
   }


  // and to use it, in client code
    var myDicList = new MultiDimDictList<string, int> ();
    myDicList.Add("ages", 23);
    myDicList.Add("ages", 32);
    myDicList.Add("ages", 18);
    myDicList.Add("salaries", 80000);
    myDicList.Add("salaries", 110000);
    myDicList.Add("accountIds", 321123);
    myDicList.Add("accountIds", 342653);

EDIT: to include an Add() method for nested instance:

public class NestedMultiDimDictList<K, K2, T>: 
           MultiDimDictList<K, MultiDimDictList<K2, T>>: 
{
       public void Add(K key, K2 key2, T addObject)
       {
           if(!ContainsKey(key)) Add(key, 
                  new MultiDimDictList<K2, T>());
           if (!base[key].Contains(key2)) 
               base[key].Add(key2, addObject);
       }    
}

AppStore - App status is ready for sale, but not in app store

We published it today after having 'Release this version'. It took 15 minutes to show on the App Store.

This is due to App Store will sync the data across servers.

Constructor of an abstract class in C#

It's there to enforce some initialization logic required by all implementations of your abstract class, or any methods you have implemented on your abstract class (not all the methods on your abstract class have to be abstract, some can be implemented).

Any class which inherits from your abstract base class will be obliged to call the base constructor.

Is there a Visual Basic 6 decompiler?

Did you try the tool named VBReFormer (http://www.decompiler-vb.net/) ? We used it a lot the past year in order to get back the source code of our application (source code we had lost 6 years ago) and it worked fine. We were also able to make some user interface changes directly from vbreformer and save them into the exe file.

What is the difference between onBlur and onChange attribute in HTML?

In Firefox the onchange fires only when you tab or else click outside the input field. The same is true of Onblur. The difference is that onblur will fire whether you changed anything in the field or not. It is possible that ENTER will fire one or both of these, but you wouldn't know that if you disable the ENTER in your forms to prevent unexpected submits.

Authorize a non-admin developer in Xcode / Mac OS

You should add yourself to the Developer Tools group. The general syntax for adding a user to a group in OS X is as follows:

sudo dscl . append /Groups/<group> GroupMembership <username>

I believe the name for the DevTools group is _developer.

Replace Div Content onclick

EDIT: This answer was submitted before the OP's jsFiddle example was posted in question. See second answer for response to that jsFiddle.


Here is an example of how it could work:

Working jsFiddle Demo

HTML:

<div id="someDiv">
    Once upon a midnight dreary
    <br>While I pondered weak and weary
    <br>Over many a quaint and curious
    <br>Volume of forgotten lore.
</div>
Type new text here:<br>
<input type="text" id="replacementtext" />
<input type="button" id="mybutt" value="Swap" />

<input type="hidden" id="vault" />

javascript/jQuery:

//Declare persistent vars outside function
var savText, newText, myState = 0;

$('#mybutt').click(function(){
    if (myState==0){
        savText = $('#someDiv').html(); //save poem data from DIV
        newText = $('#replacementtext').val(); //save data from input field
        $('#replacementtext').val(''); //clear input field
        $('#someDiv').html( newText ); //replace poem with insert field data
        myState = 1; //remember swap has been done once
    } else {
        $('#someDiv').html(savText);
        $('#replacementtext').val(newText); //replace contents
        myState = 0;
    }
});

Difference between Fact table and Dimension table?

Dimension table : It is nothing but we can maintains information about the characterized date called as Dimension table.

Example : Time Dimension , Product Dimension.

Fact Table : It is nothing but we can maintains information about the metrics or precalculation data.

Example : Sales Fact, Order Fact.

Star schema : one fact table link with dimension table form as a Start Schema.

enter image description here

How to add a filter class in Spring Boot?

From Spring docs,

Embedded servlet containers - Add a Servlet, Filter or Listener to an application

To add a Servlet, Filter, or Servlet *Listener provide a @Bean definition for it.

Eg:

@Bean
public Filter compressFilter() {
    CompressingFilter compressFilter = new CompressingFilter();
    return compressFilter;
}

Add this @Bean config to your @Configuration class and filter will be registered on startup.

Also you can add Servlets, Filters, and Listeners using classpath scanning,

@WebServlet, @WebFilter, and @WebListener annotated classes can be automatically registered with an embedded servlet container by annotating a @Configuration class with @ServletComponentScan and specifying the package(s) containing the components that you want to register. By default, @ServletComponentScan will scan from the package of the annotated class.

How to parse a JSON Input stream

{
    InputStream is = HTTPClient.get(url);
    InputStreamReader reader = new InputStreamReader(is);
    JSONTokener tokenizer = new JSONTokener(reader);
    JSONObject jsonObject = new JSONObject(tokenizer);
}

How to open, read, and write from serial port in C?

For demo code that conforms to POSIX standard as described in Setting Terminal Modes Properly and Serial Programming Guide for POSIX Operating Systems, the following is offered.
This code should execute correctly using Linux on x86 as well as ARM (or even CRIS) processors.
It's essentially derived from the other answer, but inaccurate and misleading comments have been corrected.

This demo program opens and initializes a serial terminal at 115200 baud for non-canonical mode that is as portable as possible.
The program transmits a hardcoded text string to the other terminal, and delays while the output is performed.
The program then enters an infinite loop to receive and display data from the serial terminal.
By default the received data is displayed as hexadecimal byte values.

To make the program treat the received data as ASCII codes, compile the program with the symbol DISPLAY_STRING, e.g.

 cc -DDISPLAY_STRING demo.c

If the received data is ASCII text (rather than binary data) and you want to read it as lines terminated by the newline character, then see this answer for a sample program.


#define TERMINAL    "/dev/ttyUSB0"

#include <errno.h>
#include <fcntl.h> 
#include <stdio.h>
#include <stdlib.h>
#include <string.h>
#include <termios.h>
#include <unistd.h>

int set_interface_attribs(int fd, int speed)
{
    struct termios tty;

    if (tcgetattr(fd, &tty) < 0) {
        printf("Error from tcgetattr: %s\n", strerror(errno));
        return -1;
    }

    cfsetospeed(&tty, (speed_t)speed);
    cfsetispeed(&tty, (speed_t)speed);

    tty.c_cflag |= (CLOCAL | CREAD);    /* ignore modem controls */
    tty.c_cflag &= ~CSIZE;
    tty.c_cflag |= CS8;         /* 8-bit characters */
    tty.c_cflag &= ~PARENB;     /* no parity bit */
    tty.c_cflag &= ~CSTOPB;     /* only need 1 stop bit */
    tty.c_cflag &= ~CRTSCTS;    /* no hardware flowcontrol */

    /* setup for non-canonical mode */
    tty.c_iflag &= ~(IGNBRK | BRKINT | PARMRK | ISTRIP | INLCR | IGNCR | ICRNL | IXON);
    tty.c_lflag &= ~(ECHO | ECHONL | ICANON | ISIG | IEXTEN);
    tty.c_oflag &= ~OPOST;

    /* fetch bytes as they become available */
    tty.c_cc[VMIN] = 1;
    tty.c_cc[VTIME] = 1;

    if (tcsetattr(fd, TCSANOW, &tty) != 0) {
        printf("Error from tcsetattr: %s\n", strerror(errno));
        return -1;
    }
    return 0;
}

void set_mincount(int fd, int mcount)
{
    struct termios tty;

    if (tcgetattr(fd, &tty) < 0) {
        printf("Error tcgetattr: %s\n", strerror(errno));
        return;
    }

    tty.c_cc[VMIN] = mcount ? 1 : 0;
    tty.c_cc[VTIME] = 5;        /* half second timer */

    if (tcsetattr(fd, TCSANOW, &tty) < 0)
        printf("Error tcsetattr: %s\n", strerror(errno));
}


int main()
{
    char *portname = TERMINAL;
    int fd;
    int wlen;
    char *xstr = "Hello!\n";
    int xlen = strlen(xstr);

    fd = open(portname, O_RDWR | O_NOCTTY | O_SYNC);
    if (fd < 0) {
        printf("Error opening %s: %s\n", portname, strerror(errno));
        return -1;
    }
    /*baudrate 115200, 8 bits, no parity, 1 stop bit */
    set_interface_attribs(fd, B115200);
    //set_mincount(fd, 0);                /* set to pure timed read */

    /* simple output */
    wlen = write(fd, xstr, xlen);
    if (wlen != xlen) {
        printf("Error from write: %d, %d\n", wlen, errno);
    }
    tcdrain(fd);    /* delay for output */


    /* simple noncanonical input */
    do {
        unsigned char buf[80];
        int rdlen;

        rdlen = read(fd, buf, sizeof(buf) - 1);
        if (rdlen > 0) {
#ifdef DISPLAY_STRING
            buf[rdlen] = 0;
            printf("Read %d: \"%s\"\n", rdlen, buf);
#else /* display hex */
            unsigned char   *p;
            printf("Read %d:", rdlen);
            for (p = buf; rdlen-- > 0; p++)
                printf(" 0x%x", *p);
            printf("\n");
#endif
        } else if (rdlen < 0) {
            printf("Error from read: %d: %s\n", rdlen, strerror(errno));
        } else {  /* rdlen == 0 */
            printf("Timeout from read\n");
        }               
        /* repeat read to get full message */
    } while (1);
}

For an example of an efficient program that provides buffering of received data yet allows byte-by-byte handing of the input, then see this answer.


Get root view from current activity

This is what I use to get the root view as found in the XML file assigned with setContentView:

final ViewGroup viewGroup = (ViewGroup) ((ViewGroup) this
            .findViewById(android.R.id.content)).getChildAt(0);

Change x axes scale in matplotlib

This is not so much an answer to your original question as to one of the queries you had in the body of your question.

A little preamble, so that my naming doesn't seem strange:

import matplotlib
from matplotlib import rc
from matplotlib.figure import Figure
ax = self.figure.add_subplot( 111 )

As has been mentioned you can use ticklabel_format to specify that matplotlib should use scientific notation for large or small values:

ax.ticklabel_format(style='sci',scilimits=(-3,4),axis='both')

You can affect the way that this is displayed using the flags in rcParams (from matplotlib import rcParams) or by setting them directly. I haven't found a more elegant way of changing between '1e' and 'x10^' scientific notation than:

ax.xaxis.major.formatter._useMathText = True

This should give you the more Matlab-esc, and indeed arguably better appearance. I think the following should do the same:

rc('text', usetex=True)

Android - Back button in the title bar

Light-weighted version without using ActionBarActivity that still has the same bahaviors here:

public class ToolbarConfigurer implements View.OnClickListener {
    private Activity activity;

    public ToolbarConfigurer(Activity activity, Toolbar toolbar, boolean displayHomeAsUpEnabled) {
        toolbar.setTitle((this.activity = activity).getTitle());
        if (!displayHomeAsUpEnabled) return;
        toolbar.setNavigationIcon(R.drawable.abc_ic_ab_back_mtrl_am_alpha);
        toolbar.setNavigationOnClickListener(this);
    }

    @Override
    public void onClick(View v) {
        NavUtils.navigateUpFromSameTask(activity);
    }
}

Usage: Put new ToolbarConfigurer(this, (Toolbar) findViewById(R.id.my_awesome_toolbar), true); in onCreate.

Why extend the Android Application class?

You can access variables to any class without creating objects, if its extended by Application. They can be called globally and their state is maintained till application is not killed.

Bootstrap modal opening on page load

I found the problem. This code was placed in a separate file that was added with a php include() function. And this include was happening before the Bootstrap files were loaded. So the Bootstrap JS file was not loaded yet, causing this modal to not do anything.

With the above code sample is nothing wrong and works as intended when placed in the body part of a html page.

<script type="text/javascript">
$('#memberModal').modal('show');
</script>

Is there a method to generate a UUID with go language

gofrs/uuid is the replacement for satori/go.uuid, which is the most starred UUID package for Go. It supports UUID versions 1-5 and is RFC 4122 and DCE 1.1 compliant.

import "github.com/gofrs/uuid"

// Create a Version 4 UUID, panicking on error
u := uuid.Must(uuid.NewV4())

CSS Div width percentage and padding without breaking layout

If you want the #header to be the same width as your container, with 10px of padding, you can leave out its width declaration. That will cause it to implicitly take up its entire parent's width (since a div is by default a block level element).

Then, since you haven't defined a width on it, the 10px of padding will be properly applied inside the element, rather than adding to its width:

#container {
    position: relative;
    width: 80%;
}

#header {
    position: relative;
    height: 50px;
    padding: 10px;
}

You can see it in action here.

The key when using percentage widths and pixel padding/margins is not to define them on the same element (if you want to accurately control the size). Apply the percentage width to the parent and then the pixel padding/margin to a display: block child with no width set.


Update

Another option for dealing with this is to use the box-sizing CSS rule:

#container { 
    -webkit-box-sizing: border-box; /* Safari/Chrome, other WebKit */
    -moz-box-sizing: border-box;    /* Firefox, other Gecko */
    box-sizing: border-box;         /* Opera/IE 8+ */

    /* Since this element now uses border-box sizing, the 10px of horizontal
       padding will be drawn inside the 80% width */
    width: 80%;
    padding: 0 10px;
}

Here's a post talking about how box-sizing works.

How can I add a table of contents to a Jupyter / JupyterLab notebook?

JupyterLab ToC instructions

There are already many good answers to this question, but they often require tweaks to work properly with notebooks in JupyterLab. I wrote this answer to detail the possible ways of including a ToC in a notebook while working in and exporting from JupyterLab.

As a side panel

The jupyterlab-toc extension adds the ToC as a side panel that can number headings, collapse sections, and be used for navigation (see gif below for a demo). This extension is included by default since JupyterLab 3.0, in older version you can install it with the following command

jupyter labextension install @jupyterlab/toc

enter image description here


In the notebook as a cell

At the time being, this can either be done manually as in Matt Dancho's answer, or automatically via the toc2 jupyter notebook extension in the classic notebook interface.

First, install toc2 as part of the jupyter_contrib_nbextensions bundle:

conda install -c conda-forge jupyter_contrib_nbextensions

Then, launch JupyterLab, go to Help --> Launch Classic Notebook, and open the notebook in which you want to add the ToC. Click the toc2 symbol in the toolbar to bring up the floating ToC window (see the gif below if you can't find it), click the gear icon and check the box for "Add notebook ToC cell". Save the notebook and the ToC cell will be there when you open it in JupyterLab. The inserted cell is a markdown cell with html in it, it will not update automatically.

The default options of the toc2 can be configured in the "Nbextensions" tab in the classic notebook launch page. You can e.g. choose to number headings and to anchor the ToC as a side bar (which I personally think looks cleaner).

enter image description here


In an exported HTML file

nbconvert can be used to export notebooks to HTML following rules of how to format the exported HTML. The toc2 extension mentioned above adds an export format called html_toc, which can be used directly with nbconvert from the command line (after the toc2 extension has been installed):

jupyter nbconvert file.ipynb --to html_toc
# Append `--ExtractOutputPreprocessor.enabled=False`
# to get a single html file instead of a separate directory for images

Remember that shell commands can be added to notebook cells by prefacing them with an exclamation mark !, so you can stick this line in the last cell of the notebook and always have an HTML file with a ToC generated when you hit "Run all cells" (or whatever output you desire from nbconvert). This way, you could use jupyterlab-toc to navigate the notebook while you are working, and still get ToCs in the exported output without having to resort to using the classic notebook interface (for the purists among us).

Note that configuring the default toc2 options as described above, will not change the format of nbconver --to html_toc. You need to open the notebook in the classic notebook interface for the metadata to be written to the .ipynb file (nbconvert reads the metadata when exporting) Alternatively, you can add the metadata manually via the Notebook tools tab of the JupyterLab sidebar, e.g. something like:

    "toc": {
        "number_sections": false,
        "sideBar": true
    }

If you prefer a GUI-driven approach, you should be able to open the classic notebook and click File --> Save as HTML (with ToC) (although note that this menu item was not available for me).


The gifs above are linked from the respective documentation of the extensions.

Sort a list of numerical strings in ascending order

The recommended approach in this case is to sort the data in the database, adding an ORDER BY at the end of the query that fetches the results, something like this:

SELECT temperature FROM temperatures ORDER BY temperature ASC;  -- ascending order
SELECT temperature FROM temperatures ORDER BY temperature DESC; -- descending order

If for some reason that is not an option, you can change the sorting order like this in Python:

templist = [25, 50, 100, 150, 200, 250, 300, 33]
sorted(templist, key=int)               # ascending order
> [25, 33, 50, 100, 150, 200, 250, 300]
sorted(templist, key=int, reverse=True) # descending order
> [300, 250, 200, 150, 100, 50, 33, 25]

As has been pointed in the comments, the int key (or float if values with decimals are being stored) is required for correctly sorting the data if the data received is of type string, but it'd be very strange to store temperature values as strings, if that is the case, go back and fix the problem at the root, and make sure that the temperatures being stored are numbers.

html5 - canvas element - Multiple layers

I understand that the Q does not want to use a library, but I will offer this for others coming from Google searches. @EricRowell mentioned a good plugin, but, there is also another plugin you can try, html2canvas.

In our case we are using layered transparent PNG's with z-index as a "product builder" widget. Html2canvas worked brilliantly to boil the stack down without pushing images, nor using complexities, workarounds, and the "non-responsive" canvas itself. We were not able to do this smoothly/sane with the vanilla canvas+JS.

First use z-index on absolute divs to generate layered content within a relative positioned wrapper. Then pipe the wrapper through html2canvas to get a rendered canvas, which you may leave as-is, or output as an image so that a client may save it.

How to run a .jar in mac?

Make Executable your jar and after that double click on it on Mac OS then it works successfully.

sudo chmod +x filename.jar

Try this, I hope this works.

Installing tkinter on ubuntu 14.04

If you're using Python 3 then you must install as follows:

sudo apt-get update
sudo apt-get install python3-tk

Tkinter for Python 2 (python-tk) is different from Python 3's (python3-tk).

Convert UTF-8 to base64 string

It's a little difficult to tell what you're trying to achieve, but assuming you're trying to get a Base64 string that when decoded is abcdef==, the following should work:

byte[] bytes = Encoding.UTF8.GetBytes("abcdef==");
string base64 = Convert.ToBase64String(bytes);
Console.WriteLine(base64);

This will output: YWJjZGVmPT0= which is abcdef== encoded in Base64.

Edit:

To decode a Base64 string, simply use Convert.FromBase64String(). E.g.

string base64 = "YWJjZGVmPT0=";
byte[] bytes = Convert.FromBase64String(base64);

At this point, bytes will be a byte[] (not a string). If we know that the byte array represents a string in UTF8, then it can be converted back to the string form using:

string str = Encoding.UTF8.GetString(bytes);
Console.WriteLine(str);

This will output the original input string, abcdef== in this case.

Oracle Differences between NVL and Coalesce

Though this one is obvious, and even mentioned in a way put up by Tom who asked this question. But lets put up again.

NVL can have only 2 arguments. Coalesce may have more than 2.

select nvl('','',1) from dual; //Result: ORA-00909: invalid number of arguments
select coalesce('','','1') from dual; //Output: returns 1

Extracting Path from OpenFileDialog path/filename

if (openFileDialog1.ShowDialog(this) == DialogResult.OK)
{
    strfilename = openFileDialog1.InitialDirectory + openFileDialog1.FileName;
}

display html page with node.js

Check this basic code to setup html server. its work for me.

var http = require('http'),
    fs = require('fs');


fs.readFile('./index.html', function (err, html) {
    if (err) {
        throw err; 
    }       
    http.createServer(function(request, response) {  
        response.writeHeader(200, {"Content-Type": "text/html"});  
        response.write(html);  
        response.end();  
    }).listen(8000);
});

Difference between ref and out parameters in .NET

out has gotten a new more succint syntax in C#7 https://docs.microsoft.com/en-us/dotnet/articles/csharp/whats-new/csharp-7#more-expression-bodied-members and even more exciting is the C#7 tuple enhancements that are a more elegant choice than using ref and out IMHO.

Difference between objectForKey and valueForKey?

When you do valueForKey: you need to give it an NSString, whereas objectForKey: can take any NSObject subclass as a key. This is because for Key-Value Coding, the keys are always strings.

In fact, the documentation states that even when you give valueForKey: an NSString, it will invoke objectForKey: anyway unless the string starts with an @, in which case it invokes [super valueForKey:], which may call valueForUndefinedKey: which may raise an exception.

How to display gpg key details without importing it?

To get the key IDs (8 bytes, 16 hex digits), this is the command which worked for me in GPG 1.4.16, 2.1.18 and 2.2.19:

gpg --list-packets <key.asc | awk '$1=="keyid:"{print$2}'

To get some more information (in addition to the key ID):

gpg --list-packets <key.asc

To get even more information:

gpg --list-packets -vvv --debug 0x2 <key.asc

The command

gpg --dry-run --import <key.asc

also works in all 3 versions, but in GPG 1.4.16 it prints only a short (4 bytes, 8 hex digits) key ID, so it's less secure to identify keys.

Some commands in other answers (e.g. gpg --show-keys, gpg --with-fingerprint, gpg --import --import-options show-only) don't work in some of the 3 GPG versions above, thus they are not portable when targeting multiple versions of GPG.

How to serve all existing static files directly with NGINX, but proxy the rest to a backend server.

Use try_files and named location block ('@apachesite'). This will remove unnecessary regex match and if block. More efficient.

location / {
    root /path/to/root/of/static/files;
    try_files $uri $uri/ @apachesite;

    expires max;
    access_log off;
}

location @apachesite {
    proxy_set_header X-Real-IP  $remote_addr;
    proxy_set_header Host $host;
    proxy_set_header X-Forwarded-For $proxy_add_x_forwarded_for;
    proxy_pass http://127.0.0.1:8080;
}

Update: The assumption of this config is that there doesn't exist any php script under /path/to/root/of/static/files. This is common in most modern php frameworks. In case your legacy php projects have both php scripts and static files mixed in the same folder, you may have to whitelist all of the file types you want nginx to serve.

What is the difference between .NET Core and .NET Standard Class Library project types?

Another way of explaining the difference could be with real world examples, as most of us mere mortals will use existing tools and frameworks (Xamarin, Unity, etc.) to do the job.

So, with .NET Framework you have all the .NET tools to work with, but you can only target Windows applications (UWP, Windows Forms, ASP.NET, etc.). Since .NET Framework is closed source there isn't much to do about it.

With .NET Core you have fewer tools, but you can target the main desktop platforms (Windows, Linux, and Mac). This is specially useful in ASP.NET Core applications, since you can now host ASP.NET on Linux (cheaper hosting prices). Now, since .NET Core was open sourced, it's technically possible to develop libraries for other platforms. But since there aren't frameworks that support it, I don't think that's a good idea.

With .NET Standard you have even fewer tools, but you can target all/most platforms. You can target mobile thanks to Xamarin, and you can even target game consoles thanks to Mono/Unity. It's also possible to target web clients with the UNO platform and Blazor (although both are kind of experimental right now).

In a real-world application you may need to use all of them. For example, I developed a point of sale application that had the following architecture:

Shared both server and slient:

  • A .NET Standard library that handles the models of my application.
  • A .NET Standard library that handles the validation of data sent by the clients.

Since it's a .NET Standard library, it can be used in any other project (client and server).

Also a nice advantage of having the validation on a .NET standard library since I can be sure the same validation is applied on the server and the client. Server is mandatory, while client is optional and useful to reduce traffic.

Server side (Web API):

  • A .NET Standard (could be .NET Core as well) library that handles all the database connections.

  • A .NET Core project that handles the Rest API and makes use of the database library.

As this is developed in .NET Core, I can host the application on a Linux server.

Client side (MVVM with WPF + Xamarin.Forms Android/iOS):

  • A .NET Standard library that handles the client API connection.

  • A .NET Standard library that handles the ViewModels logic. It is used in all the views.

  • A .NET Framework WPF application that handles the WPF views for a windows application. WPF applications can be .NET core now, although they only work on Windows currently. AvaloniaUI is a good alternative for making desktop GUI applications for other desktop platforms.

  • A .NET Standard library that handles Xamarin forms views.

  • A Xamarin Android and Xamarin iOS project.

So you can see that there's a big advantage here on the client side of the application, since I can reuse both .NET Standard libraries (client API and ViewModels) and just make views with no logic for the WPF, Xamarin and iOS applications.

PHP: How do you determine every Nth iteration of a loop?

You can also do it without modulus. Just reset your counter when it matches.

if($counter == 2) { // matches every 3 iterations
   echo 'image-file';
   $counter = 0; 
}

How to set focus on an input field after rendering?

You don't need getInputDOMNode?? in this case...

Just simply get the ref and focus() it when component gets mounted -- componentDidMount...

import React from 'react';
import { render } from 'react-dom';

class myApp extends React.Component {

  componentDidMount() {
    this.nameInput.focus();
  }

  render() {
    return(
      <div>
        <input ref={input => { this.nameInput = input; }} />
      </div>
    );
  }

}

ReactDOM.render(<myApp />, document.getElementById('root'));

XPath: difference between dot and text()

There is a difference between . and text(), but this difference might not surface because of your input document.

If your input document looked like (the simplest document one can imagine given your XPath expressions)

Example 1

<html>
  <a>Ask Question</a>
</html>

Then //a[text()="Ask Question"] and //a[.="Ask Question"] indeed return exactly the same result. But consider a different input document that looks like

Example 2

<html>
  <a>Ask Question<other/>
  </a>
</html>

where the a element also has a child element other that follows immediately after "Ask Question". Given this second input document, //a[text()="Ask Question"] still returns the a element, while //a[.="Ask Question"] does not return anything!


This is because the meaning of the two predicates (everything between [ and ]) is different. [text()="Ask Question"] actually means: return true if any of the text nodes of an element contains exactly the text "Ask Question". On the other hand, [.="Ask Question"] means: return true if the string value of an element is identical to "Ask Question".

In the XPath model, text inside XML elements can be partitioned into a number of text nodes if other elements interfere with the text, as in Example 2 above. There, the other element is between "Ask Question" and a newline character that also counts as text content.

To make an even clearer example, consider as an input document:

Example 3

<a>Ask Question<other/>more text</a>

Here, the a element actually contains two text nodes, "Ask Question" and "more text", since both are direct children of a. You can test this by running //a/text() on this document, which will return (individual results separated by ----):

Ask Question
-----------------------
more text

So, in such a scenario, text() returns a set of individual nodes, while . in a predicate evaluates to the string concatenation of all text nodes. Again, you can test this claim with the path expression //a[.='Ask Questionmore text'] which will successfully return the a element.


Finally, keep in mind that some XPath functions can only take one single string as an input. As LarsH has pointed out in the comments, if such an XPath function (e.g. contains()) is given a sequence of nodes, it will only process the first node and silently ignore the rest.

JavaScript hard refresh of current page

Try to use:

location.reload(true);

When this method receives a true value as argument, it will cause the page to always be reloaded from the server. If it is false or not specified, the browser may reload the page from its cache.

More info:

Push to GitHub without a password using ssh-key

Additionally for gists, it seems you must leave out the username

git remote set-url origin [email protected]:<Project code>

Accessing Arrays inside Arrays In PHP

You can access the inactive tags array with (assuming $myArray contains the array)

$myArray['inactiveTags'];

Your question doesn't seem to go beyond accessing the contents of the inactiveTags key so I can only speculate with what your final goal is.

The first key:value pair in the inactiveTags array is

array ('195' => array(
                 'id' => 195, 
                 'tag' => 'auto')
      )

To access the tag value, you would use

$myArray['inactiveTags'][195]['tag']; // auto

If you want to loop through each inactiveTags element, I would suggest:

foreach($myArray['inactiveTags'] as $value) {
  print $value['id'];
  print $value['tag'];
}

This will print all the id and tag values for each inactiveTag

Edit:: For others to see, here is a var_dump of the array provided in the question since it has not readible

array
  'languages' => 
    array
      76 => 
        array
          'id' => string '76' (length=2)
          'tag' => string 'Deutsch' (length=7)
  'targets' => 
    array
      81 => 
        array
          'id' => string '81' (length=2)
          'tag' => string 'Deutschland' (length=11)
  'tags' => 
    array
      7866 => 
        array
          'id' => string '7866' (length=4)
          'tag' => string 'automobile' (length=10)
      17800 => 
        array
          'id' => string '17800' (length=5)
          'tag' => string 'seat leon' (length=9)
      17801 => 
        array
          'id' => string '17801' (length=5)
          'tag' => string 'seat leon cupra' (length=15)
  'inactiveTags' => 
    array
      195 => 
        array
          'id' => string '195' (length=3)
          'tag' => string 'auto' (length=4)
      17804 => 
        array
          'id' => string '17804' (length=5)
          'tag' => string 'coupès' (length=6)
      17805 => 
        array
          'id' => string '17805' (length=5)
          'tag' => string 'fahrdynamik' (length=11)
      901 => 
        array
          'id' => string '901' (length=3)
          'tag' => string 'fahrzeuge' (length=9)
      17802 => 
        array
          'id' => string '17802' (length=5)
          'tag' => string 'günstige neuwagen' (length=17)
      1991 => 
        array
          'id' => string '1991' (length=4)
          'tag' => string 'motorsport' (length=10)
      2154 => 
        array
          'id' => string '2154' (length=4)
          'tag' => string 'neuwagen' (length=8)
      10660 => 
        array
          'id' => string '10660' (length=5)
          'tag' => string 'seat' (length=4)
      17803 => 
        array
          'id' => string '17803' (length=5)
          'tag' => string 'sportliche ausstrahlung' (length=23)
      74 => 
        array
          'id' => string '74' (length=2)
          'tag' => string 'web 2.0' (length=7)
  'categories' => 
    array
      16082 => 
        array
          'id' => string '16082' (length=5)
          'tag' => string 'Auto & Motorrad' (length=15)
      51 => 
        array
          'id' => string '51' (length=2)
          'tag' => string 'Blogosphäre' (length=11)
      66 => 
        array
          'id' => string '66' (length=2)
          'tag' => string 'Neues & Trends' (length=14)
      68 => 
        array
          'id' => string '68' (length=2)
          'tag' => string 'Privat' (length=6)

Prevent browser caching of AJAX call result

another way is to provide no cache headers from serverside in the code that generates the response to ajax call:

response.setHeader( "Pragma", "no-cache" );
response.setHeader( "Cache-Control", "no-cache" );
response.setDateHeader( "Expires", 0 );

Convert hex string to int in Python

The formatter option '%x' % seems to work in assignment statements as well for me. (Assuming Python 3.0 and later)

Example

a = int('0x100', 16)
print(a)   #256
print('%x' % a) #100
b = a
print(b) #256
c = '%x' % a
print(c) #100

Difference between 2 dates in seconds

$timeFirst  = strtotime('2011-05-12 18:20:20');
$timeSecond = strtotime('2011-05-13 18:20:20');
$differenceInSeconds = $timeSecond - $timeFirst;

You will then be able to use the seconds to find minutes, hours, days, etc.

Find out if string ends with another string in C++

you can use string::rfind

The full Example based on comments:

bool EndsWith(string &str, string& key)
{
size_t keylen = key.length();
size_t strlen = str.length();

if(keylen =< strlen)
    return string::npos != str.rfind(key,strlen - keylen, keylen);
else return false;
}

How to perform update operations on columns of type JSONB in Postgres 9.4

I wrote small function for myself that works recursively in Postgres 9.4. I had same problem (good they did solve some of this headache in Postgres 9.5). Anyway here is the function (I hope it works well for you):

CREATE OR REPLACE FUNCTION jsonb_update(val1 JSONB,val2 JSONB)
RETURNS JSONB AS $$
DECLARE
    result JSONB;
    v RECORD;
BEGIN
    IF jsonb_typeof(val2) = 'null'
    THEN 
        RETURN val1;
    END IF;

    result = val1;

    FOR v IN SELECT key, value FROM jsonb_each(val2) LOOP

        IF jsonb_typeof(val2->v.key) = 'object'
            THEN
                result = result || jsonb_build_object(v.key, jsonb_update(val1->v.key, val2->v.key));
            ELSE
                result = result || jsonb_build_object(v.key, v.value);
        END IF;
    END LOOP;

    RETURN result;
END;
$$ LANGUAGE plpgsql;

Here is sample use:

select jsonb_update('{"a":{"b":{"c":{"d":5,"dd":6},"cc":1}},"aaa":5}'::jsonb, '{"a":{"b":{"c":{"d":15}}},"aa":9}'::jsonb);
                            jsonb_update                             
---------------------------------------------------------------------
 {"a": {"b": {"c": {"d": 15, "dd": 6}, "cc": 1}}, "aa": 9, "aaa": 5}
(1 row)

As you can see it analyze deep down and update/add values where needed.

Delete last char of string

string.Join is better, but if you really want a LINQ ForEach:

var strgroupids = string.Empty;

groupIds.ForEach(g =>
{
    if(strgroupids != string.Empty){
        strgroupids += ",";
    }

    strgroupids += g;
});

Some notes:

  • string.Join and foreach are both better than this, vastly slower, approach
  • No need to remove the last , since it's never appended
  • The increment operator (+=) is handy for appending to strings
  • .ToString() is unnecessary as it is called automatically when concatenating non-strings
  • When handling large strings, StringBuilder should be considered instead of concatenating strings

Launch programs whose path contains spaces

It's working with

Set WSHELL = CreateObject("Wscript.Shell")
WSHELL.Exec("Application_Path")

But what should be the parameter in case we want to enter the application name only

e.g in case of Internet Explorer

WSHELL.Run("iexplore")

Unzip files (7-zip) via cmd command

In Windows 10 I had to run the batch file as an administrator.

Which language uses .pde extension?

The .pde file extension is the one used by the Processing, Wiring, and the Arduino IDE.

Processing is not C-based but rather Java-based and with a syntax derived from Java. It is a Java framework that can be used as a Java library. It includes a default IDE that uses .pde extension. Just wanted to rectify @kersny's answer.

Wiring is a microcontroller that uses the same IDE. Arduino uses a modified version, but also with .pde. The OpenProcessing page where you found it is a website to exhibit some Processing work.

If you know Java, it should be fairly easy to convert the Processing code to Java AWT.

jQuery: Clearing Form Inputs

You may try

$("#addRunner input").each(function(){ ... });

Inputs are no selectors, so you do not need the : Haven't tested it with your code. Just a fast guess!

git submodule tracking latest

Edit (2020.12.28): GitHub change default master branch to main branch since October 2020. See https://github.com/github/renaming


Update March 2013

Git 1.8.2 added the possibility to track branches.

"git submodule" started learning a new mode to integrate with the tip of the remote branch (as opposed to integrating with the commit recorded in the superproject's gitlink).

# add submodule to track master branch
git submodule add -b master [URL to Git repo];

# update your submodule
git submodule update --remote 

If you had a submodule already present you now wish would track a branch, see "how to make an existing submodule track a branch".

Also see Vogella's tutorial on submodules for general information on submodules.

Note:

git submodule add -b . [URL to Git repo];
                    ^^^

See git submodule man page:

A special value of . is used to indicate that the name of the branch in the submodule should be the same name as the current branch in the current repository.


See commit b928922727d6691a3bdc28160f93f25712c565f6:

submodule add: If --branch is given, record it in .gitmodules

This allows you to easily record a submodule.<name>.branch option in .gitmodules when you add a new submodule. With this patch,

$ git submodule add -b <branch> <repository> [<path>]
$ git config -f .gitmodules submodule.<path>.branch <branch>

reduces to

$ git submodule add -b <branch> <repository> [<path>]

This means that future calls to

$ git submodule update --remote ...

will get updates from the same branch that you used to initialize the submodule, which is usually what you want.

Signed-off-by: W. Trevor King [email protected]


Original answer (February 2012):

A submodule is a single commit referenced by a parent repo.
Since it is a Git repo on its own, the "history of all commits" is accessible through a git log within that submodule.

So for a parent to track automatically the latest commit of a given branch of a submodule, it would need to:

  • cd in the submodule
  • git fetch/pull to make sure it has the latest commits on the right branch
  • cd back in the parent repo
  • add and commit in order to record the new commit of the submodule.

gitslave (that you already looked at) seems to be the best fit, including for the commit operation.

It is a little annoying to make changes to the submodule due to the requirement to check out onto the correct submodule branch, make the change, commit, and then go into the superproject and commit the commit (or at least record the new location of the submodule).

Other alternatives are detailed here.

Is there a way to make mv create the directory to be moved to if it doesn't exist?

This will move foo.c to the new directory baz with the parent directory bar.

mv foo.c `mkdir -p ~/bar/baz/ && echo $_`

The -p option to mkdir will create intermediate directories as required.
Without -p all directories in the path prefix must already exist.

Everything inside backticks `` is executed and the output is returned in-line as part of your command.
Since mkdir doesn't return anything, only the output of echo $_ will be added to the command.

$_ references the last argument to the previously executed command.
In this case, it will return the path to your new directory (~/bar/baz/) passed to the mkdir command.


I unzipped an archive without giving a destination and wanted to move all the files except demo-app.zip from my current directory to a new directory called demo-app.
The following line does the trick:

mv `ls -A | grep -v demo-app.zip` `mkdir -p demo-app && echo $_`

ls -A returns all file names including hidden files (except for the implicit . and ..).

The pipe symbol | is used to pipe the output of the ls command to grep (a command-line, plain-text search utility).
The -v flag directs grep to find and return all file names excluding demo-app.zip.
That list of files is added to our command-line as source arguments to the move command mv. The target argument is the path to the new directory passed to mkdir referenced using $_ and output using echo.

How to find a min/max with Ruby

All those results generate garbage in a zealous attempt to handle more than two arguments. I'd be curious to see how they perform compared to good 'ol:

def max (a,b)
  a>b ? a : b
end

which is, by-the-way, my official answer to your question.

How to find the Target *.exe file of *.appref-ms

ClickOnce apps are designed so that the end user downloads a "downloader" - the ClickOnce app, then when ya run it, it downloads and installs in %LocalAppData%\Apps\2.0..... and then it's random folder names for every OS install you do. Backing up is pointless and so is trying to move the program. The point of ClickOnce is 2-Fold: 1. AutoUpdating of the program 2. The end user has no installer and also can't move the app or it breaks

The %LocalAppData%\Apps\2.0..... folder is the program AND %LocalAppData%\GitHub is the settings folder.

I'm not going to cover how to circumvent this - only stating the above. :P

The best 'tip' I can say legitimately is: You 'can' in some cases move the final folder that all the files are in and use a symlink back, if you are low on space. But, not all apps will work and essentially will delete the symlink once you they run. Then they might reinstall or simply just remove the link. Keep in mind also, other apps may be using that same final folder as well, so move the folder will affect those too.

REST response code for invalid data

400 is the best choice in both cases. If you want to further clarify the error you can either change the Reason Phrase or include a body to explain the error.

412 - Precondition failed is used for conditional requests when using last-modified date and ETags.

403 - Forbidden is used when the server wishes to prevent access to a resource.

The only other choice that is possible is 422 - Unprocessable entity.

C# list.Orderby descending

Yes. Use OrderByDescending instead of OrderBy.

Button Listener for button in fragment in android

Your fragment class should implement OnClickListener

public class SmartTvControllerFragment extends Fragment implements View.OnClickListener

Then get view, link button and set onClickListener like in example below

 View view;

@Override
public View onCreateView(LayoutInflater inflater, ViewGroup container, Bundle savedInstanceState) {

    view = inflater.inflate(R.layout.smart_tv_controller_fragment, container, false);
    upButton = (Button) view.findViewById(R.id.smart_tv_controller_framgment_up_button);
    upButton.setOnClickListener(this);
    return view;
 }

And then add onClickListener method and do what you want.

@Override
public void onClick(View v) {
 //do what you want to do when button is clicked
    switch (v.getId()) {
        case R.id.textView_help:
            switchFragment(HelpFragment.TAG);
            break;
        case R.id.textView_settings:
            switchFragment(SettingsFragment.TAG);
            break;
    }
}

This is my example of code, but I hope you understood

Android Studio Could not initialize class org.codehaus.groovy.runtime.InvokerHelper

I also had the same problem and and it looks like this problem is due to the gradle version in the project directory

  • This is the version of my jdk:
openjdk version "15-ea" 2020-09-15
OpenJDK Runtime Environment (build 15-ea+32-Ubuntu-220.04)
OpenJDK 64-Bit Server VM (build 15-ea+32-Ubuntu-220.04, mixed mode, sharing)
  • Please check gradle whether it is installed. This is the version of my gradle:
------------------------------------------------------------
Gradle 6.8
------------------------------------------------------------

Build time:   2021-01-08 16:38:46 UTC
Revision:     b7e82460c5373e194fb478a998c4fcfe7da53a7e

Kotlin:       1.4.20
Groovy:       2.5.12
Ant:          Apache Ant(TM) version 1.10.9 compiled on September 27 2020
JVM:          15-ea (Private Build 15-ea+32-Ubuntu-220.04)
OS:           Linux 5.4.0-65-generic amd64
  • Then open the gradle-wrapper.properties file in your project folder, in the folder: project name/android/gradle/wrapper/gradle-wrapper.properties

  • Change the gradle version. Here I changed the gradle version to 6.5

The video version can be seen here: https://www.youtube.com/watch?v=mcclcscUpV0

I get "Http failure response for (unknown url): 0 Unknown Error" instead of actual error message in Angular

For me it wasn't an angular problem. Was a field of type DateTime in the DB that has a value of (0000-00-00) and my model cannot bind that property correct so I changed to a valid value like (2019-08-12).

I'm using .net core, OData v4 and MySql (EF pomelo connector)

Do subclasses inherit private fields?

For example,

class Person {
    private String name;

    public String getName () {
        return this.name;
    }

    Person(String name) {
        this.name = name;
    }
}
public class Student extends Person {

    Student(String name) {
        super(name);
    }
    
    public String getStudentName() {
        return this.getName(); // works
        // "return this.name;" doesn't work, and the error is "The field Person.name is not visible"

    }
}

public class Main {
    public static void main(String[] args) {
        Student s = new Student("Bill");

        String name = s.getName(); // works
        // "String name = s.name;" doesn't work, and the error is "The field Person.name is not visible"

        System.out.println(name);
    }
}

Difference between Activity and FragmentActivity

A FragmentActivity is a subclass of Activity that was built for the Android Support Package.

The FragmentActivity class adds a couple new methods to ensure compatibility with older versions of Android, but other than that, there really isn't much of a difference between the two. Just make sure you change all calls to getLoaderManager() and getFragmentManager() to getSupportLoaderManager() and getSupportFragmentManager() respectively.

What is the difference between an interface and abstract class?

The general idea of abstract classes and interfaces is to be extended/implemented by other classes (cannot be constructed alone) that use these general "settings" (some kind of a template), making it simple to set a specific-general behaviour for all the objects that later extend it.

An abstract class has regular methods set AND abstract methods. Extended classes can include unset methods after being extended by an abstract class. When setting abstract methods - they are defined by the classes that are extending it later.

Interfaces have the same properties as an abstract class, but includes only abstract methods, which could be implemented in an other class/es (and can be more than one interface to implement), this creates a more permanent-solid definishion of methods/static variables. Unlike the abstract class, you cannot add custom "regular" methods.

Is HTML considered a programming language?

Well, L is for language, but it doesn't imply programming language. After all, English or French are (natural) languages too! ;-)

As said above, put them under a subsidiary section, Technology seems to be a good term.

(Looking at my own resume, not updated in a while) I have made a section just called "Languages", so I can't get wrong... :-D
I have put "(X)HTML and CSS, XML/DTD/Schema and SVG" at the end of the section, clearly separated.

In French, I have a section "Langages" (programming and markup) and another "Langues" (French/English). In the English version, I titled both at "Languages", which is clumsy now that I think of it, although context clarify this. I should find a better formulation.

Five equal columns in twitter bootstrap

For twitter bootstrap 3 this is the most simple way to achieve this:

<section class="col col-sm-3" style="width: 20%;">
<section class="col col-sm-3" style="width: 20%;">
<section class="col col-sm-3" style="width: 20%;">
<section class="col col-sm-3" style="width: 20%;">
<section class="col col-sm-3" style="width: 20%;">

The definitive guide to form-based website authentication

I dont't know whether it was best to answer this as an answer or as a comment. I opted for the first option.

Regarding the poing PART IV: Forgotten Password Functionality in the first answer, I would make a point about Timing Attacks.

In the Remember your password forms, an attacker could potentially check a full list of emails and detect which are registered to the system (see link below).

Regarding the Forgotten Password Form, I would add that it is a good idea to equal times between successful and unsucessful queries with some delay function.

https://crypto.stanford.edu/~dabo/papers/webtiming.pdf

How to find out line-endings in a text file?

You can use xxd to show a hex dump of the file, and hunt through for "0d0a" or "0a" chars.

You can use cat -v <filename> as @warriorpostman suggests.

Change status bar color with AppCompat ActionBarActivity

I don't think the status bar color has been implemented in AppCompat yet. These are the attributes which are available:

    <!-- ============= -->
    <!-- Color palette -->
    <!-- ============= -->

    <!-- The primary branding color for the app. By default, this is the color applied to the
         action bar background. -->
    <attr name="colorPrimary" format="color" />

    <!-- Dark variant of the primary branding color. By default, this is the color applied to
         the status bar (via statusBarColor) and navigation bar (via navigationBarColor). -->
    <attr name="colorPrimaryDark" format="color" />

    <!-- Bright complement to the primary branding color. By default, this is the color applied
         to framework controls (via colorControlActivated). -->
    <attr name="colorAccent" format="color" />

    <!-- The color applied to framework controls in their normal state. -->
    <attr name="colorControlNormal" format="color" />

    <!-- The color applied to framework controls in their activated (ex. checked) state. -->
    <attr name="colorControlActivated" format="color" />

    <!-- The color applied to framework control highlights (ex. ripples, list selectors). -->
    <attr name="colorControlHighlight" format="color" />

    <!-- The color applied to framework buttons in their normal state. -->
    <attr name="colorButtonNormal" format="color" />

    <!-- The color applied to framework switch thumbs in their normal state. -->
    <attr name="colorSwitchThumbNormal" format="color" />

(From \sdk\extras\android\support\v7\appcompat\res\values\attrs.xml)

What's the proper value for a checked attribute of an HTML checkbox?

I think this may help:


First read all the specs from Microsoft and W3.org.
You'd see that the setting the actual element of a checkbox needs to be done on the ELEMENT PROPERTY, not the UI or attribute.
$('mycheckbox')[0].checked

Secondly, you need to be aware that the checked attribute RETURNS a string "true", "false"
Why is this important? Because you need to use the correct Type. A string, not a boolean. This also important when parsing your checkbox.
$('mycheckbox')[0].checked = "true"

if($('mycheckbox')[0].checked === "true"){ //do something }

You also need to realize that the "checked" ATTRIBUTE is for setting the value of the checkbox initially. This doesn't do much once the element is rendered to the DOM. Picture this working when the webpage loads and is initially parsed.
I'll go with IE's preference on this one: <input type="checkbox" checked="checked"/>

Lastly, the main aspect of confusion for a checkbox is that the checkbox UI element is not the same as the element's property value. They do not correlate directly. If you work in .net, you'll discover that the user "checking" a checkbox never reflects the actual bool value passed to the controller. To set the UI, I use both $('mycheckbox').val(true); and $('mycheckbox').attr('checked', 'checked');


In short, for a checked checkbox you need:
Initial DOM: <input type="checkbox" checked="checked">
Element Property: $('mycheckbox')[0].checked = "true";
UI: $('mycheckbox').val(true); and $('mycheckbox').attr('checked', 'checked');

Meaning of @classmethod and @staticmethod for beginner?

Though classmethod and staticmethod are quite similar, there's a slight difference in usage for both entities: classmethod must have a reference to a class object as the first parameter, whereas staticmethod can have no parameters at all.

Example

class Date(object):

    def __init__(self, day=0, month=0, year=0):
        self.day = day
        self.month = month
        self.year = year

    @classmethod
    def from_string(cls, date_as_string):
        day, month, year = map(int, date_as_string.split('-'))
        date1 = cls(day, month, year)
        return date1

    @staticmethod
    def is_date_valid(date_as_string):
        day, month, year = map(int, date_as_string.split('-'))
        return day <= 31 and month <= 12 and year <= 3999

date2 = Date.from_string('11-09-2012')
is_date = Date.is_date_valid('11-09-2012')

Explanation

Let's assume an example of a class, dealing with date information (this will be our boilerplate):

class Date(object):

    def __init__(self, day=0, month=0, year=0):
        self.day = day
        self.month = month
        self.year = year

This class obviously could be used to store information about certain dates (without timezone information; let's assume all dates are presented in UTC).

Here we have __init__, a typical initializer of Python class instances, which receives arguments as a typical instancemethod, having the first non-optional argument (self) that holds a reference to a newly created instance.

Class Method

We have some tasks that can be nicely done using classmethods.

Let's assume that we want to create a lot of Date class instances having date information coming from an outer source encoded as a string with format 'dd-mm-yyyy'. Suppose we have to do this in different places in the source code of our project.

So what we must do here is:

  1. Parse a string to receive day, month and year as three integer variables or a 3-item tuple consisting of that variable.
  2. Instantiate Date by passing those values to the initialization call.

This will look like:

day, month, year = map(int, string_date.split('-'))
date1 = Date(day, month, year)

For this purpose, C++ can implement such a feature with overloading, but Python lacks this overloading. Instead, we can use classmethod. Let's create another "constructor".

    @classmethod
    def from_string(cls, date_as_string):
        day, month, year = map(int, date_as_string.split('-'))
        date1 = cls(day, month, year)
        return date1

date2 = Date.from_string('11-09-2012')

Let's look more carefully at the above implementation, and review what advantages we have here:

  1. We've implemented date string parsing in one place and it's reusable now.
  2. Encapsulation works fine here (if you think that you could implement string parsing as a single function elsewhere, this solution fits the OOP paradigm far better).
  3. cls is an object that holds the class itself, not an instance of the class. It's pretty cool because if we inherit our Date class, all children will have from_string defined also.

Static method

What about staticmethod? It's pretty similar to classmethod but doesn't take any obligatory parameters (like a class method or instance method does).

Let's look at the next use case.

We have a date string that we want to validate somehow. This task is also logically bound to the Date class we've used so far, but doesn't require instantiation of it.

Here is where staticmethod can be useful. Let's look at the next piece of code:

    @staticmethod
    def is_date_valid(date_as_string):
        day, month, year = map(int, date_as_string.split('-'))
        return day <= 31 and month <= 12 and year <= 3999

    # usage:
    is_date = Date.is_date_valid('11-09-2012')

So, as we can see from usage of staticmethod, we don't have any access to what the class is---it's basically just a function, called syntactically like a method, but without access to the object and its internals (fields and another methods), while classmethod does.

Swing vs JavaFx for desktop applications

I don't think there's any one right answer to this question, but my advice would be to stick with SWT unless you are encountering severe limitations that require such a massive overhaul.

Also, SWT is actually newer and more actively maintained than Swing. (It was originally developed as a replacement for Swing using native components).

How do I display todays date on SSRS report?

You can place a text-box to the report and add an expression with the following value in it:

="Report generation date:  " & Format(Globals!ExecutionTime,"dd/MM/yyyy  h:mm:ss tt" )

Exception : AAPT2 error: check logs for details

For me, I got this error while working on some Udacity projects. I fixed it by adding the following code to the top-level build.gradle file.

allprojects {
    String osName = System.getProperty("os.name").toLowerCase()
    if (osName.contains("windows")) {
        buildDir = "C:/tmp/${rootProject.name}/${project.name}"
    }
    repositories {
        jcenter()
        google()
    }
}